Download as pdf or txt
Download as pdf or txt
You are on page 1of 50

CHAPTER

/
r * )) /
1.1

Theory of Equations
Learning Part
Session 1
• Polynomial in One Variable • Identity
• Linear Equation • Quadratic Equations
• Standard Quadratic Equation
Session 2
• Transformation of Quadratic Equations • Condition for Common Roots
Session 3
• Quadratic Expression
• Wavy Curve Method
• Condition for Resolution into Linear Factors
• Location of Roots (Interval in which Roots Lie)
Session 4
• Equations of Higher Degree
• Rational Algebraic Inequalities
• Roots of Equation with the Help of Graphs
Session 5
• Irrational Equations
• Irrational Inequations
• Exponential Equations
• Exponential Inequations
• Logarithmic Equations
• Logarithmic Inequations

Practice Part
• JEE Type Examples
• Chapter Exercises

; Arihant on Your Mobile!


Exercises with the § symbol can be practised on your mobile. See inside cover page to activate for free.
Session 1
Polynomial in One Variable, Identity, Linear Equation,
Quadratic Equations, Standard Quadratic Equation
Polynomial in One Variable Thus, every polynomial is a rational expression but a
An algebraic expression containing many terms of the rational expression may or may not be a polynomial.
For example,
form ex", n being a non-negative integer is called a
2
polynomial, (i) x2 -7x+8 (ii)
x-3
i.e., f(x) = a0 -x" + a, -x"’1 + a2 • x" 2
x3 -6x2 + llx -6 3 or x2 +3
/• x x + —
+ ... +an_! -x + an, (iii) (iv)
(x-4) x x
where x is a variable, a0,a},a2,...,an are constants and
o0 £0.
4. Degree of Polynomial
1. Real Polynomial The highest power of variable (x) present in the
polynomial is called the degree of the polynomial.
Let a0, a!, a2,.... an be real numbers and x is a real variable.
For example, f(x) = a0 • xn +aj • x"-1 +a2 • xn~2
Then,
+... + an _j ■ x + an is a polynomial in x of degree n.
/(x) = a0 • x"n + flj • x"'1 +a+ a2? •■ x"
xn~2 + ... +an-i • x + an
Remark
is called a real polynomial of real variable (x) with real A polynomial of degree one is generally called a linear
coefficients. polynomial. Polynomials of degree 2,3,4 and 5 are known as
For example, 5x3 -3x2 + 7x - 4,x2 -3x +1, etc., are real quadratic, cubic, biquadratic and pentic polynomials,
respectively.
polynomials.
5. Polynomial Equation
2. Complex Polynomial If f(x) is a polynomial, real or complex, then /(x) =0 is
Let a0,aj,a2,...,an are complex numbers and x is a called a polynomial equation.
varying complex number.
(i) A polynomial equation has atleast one root.
Then /(x) = a0- x"+ a, - x”-1 +a2- xn-2+...+ an_] • x+a„
(ii) A polynomial equation of degree n has n roots.
is called a complex polynomial or a polynomial of complex
variable with complex coefficients. Remarks
For example, x3 -7ix2 + (3-2i)x + 13,3x2 -(2+ 3i)x+5i, 1. A polynomial equation of degree one is called a linear
equation i.e. ax + b = 0, where a, b e C, set of all complex
etc. (where i = V-l) are complex polynomials. numbers and a * 0.
2. A polynomial equation of degree two is called a quadratic
equation i.e., ax2 + bx + c, where a, b, c e Cand a * 0.
3. Rational Expression 3. A polynomial equation of degree three is called a cubic
or Rational Function equation i.e., ax3 + bx2 + ex + d - 0, where a, b, c, d e Gand
P(x) a*0.
An expression of the form------ , where P(x) and Q(x) 4. A polynomial equation of degree four is called a biquadratic
Q(x) equation i.e., ax4 + bx3 + ex2 + dx + e = 0, where
are polynomials in x, is called a rational expression. As a a, b.c.d.eeCand a * 0.
particular case when Q(x) is a non-zero constant 5. A polynomial equation of degree five is called a pentic
Q(x) equation i.e., ax5 + bxA + ex3 + dx2 + ex + f = 0, where
reduces to a polynomial. a, b,c,d,e,f eC and a * 0.
Chap 02 Theory of Equations 105

6. Roots of an Equation I Example 1. If equation


(X2 -5X + 6)x2 + (X2 -3X + 2)x + (X2 -4) = 0 is
The values of the variable for which an equation is
satisfied are called the roots of the equation. satisfied by more than two values of x, find the
If x = a is a root of the equation /(x) =0, then /(a) =0. parameter X.
Sol. If an equation of degree two is satisfied by more than two
Remark values of unknown, then it must be an identity. Then, we
The real roots of an equation f(x) = 0 are the values of x, where must have
the curve y = f(x) crosses /-axis. X2 - 5X + 6 = 0, X2 - 3X + 2 = 0, X2 - 4 = 0
1 = 2,3 and A, = 2, land X = 2,-2
7. Solution Set Common value of X which satisfies each condition is X = 2.
The set of all roots of an equation in a given domain is
called the solution set of the equation.
I Example 2. Show that
(x + b)(x + c) (x + c)(x + o) + (x + o)(x + b)
For example, The roots of the equation (b - a) (c - a)+ (c - b) (a - b)+ (a - c) (b - c)
x3 -2x2 -5x +6 = 0 are 1, -2,3, the solution set is
{1,-2,3}. is an identity.
Sol. Given relation is
Remark (x + b)(x + c) + (x + c)(x + a) + (x + a)(x + fr) _ 1
Solve or solving an equation means finding its solution set or
obtaining all its roots.
(b-a)(c-a) (c - b)(a- b) (a-c)(b-c)

When x = - a, then LHS of Eq. (i) = ———— = 1


(b - a) (c - a)
Identity = RHS of Eq. (i)
If two expressions are equal for all values of x, then the When x = - b, then LHS of Eq. (i)
statement of equality between the two expressions is = <c ~ =j= rhs of E
called an identity. (c~b)(a-b)
For example, (x +1)2 = x2 + 2x +1 is an identity in x. (a — c)(b — c)
and when x = - c, then LHS of Eq. (i) =--------------- = 1
(a-c)(b-c)
or
= RHS of Eq. (i).
If f(x) =0 is satisfied by every value of x in the domain of
Thus, highest power of x occurring in relation of Eq. (i) is 2
/(x), then it is called an identity.
and this relation is satisfied by three distinct values of
For example, f(x)=(x +1)2 -(x2 + 2x +1) =0 is an x (= - a, - b, - c). Therefore, it cannot be an equation and
identity in the domain C, as it is satisfied by every hence it is an identity.
complex number.
I Example 3. Show that x2 - 3| x| + 2 = 0 is an
or
n equation.
Iff(x) = a,0 • x" +a, ■ x"-1 + a2 • xn~2
Sol. Put x = 0 in x2 - 3 |x | + 2 = 0
+ ... + un_i[ • x + an =0 have more than n distinct roots, it
is an identity, then => 02-3|0| + 2 = 2*0
a0 -<2i =a2 = ...=an_1 =an =0 Since, the relation x2 - 3 |x| + 2 = 0 is not satisfied by x = 0.
For example, If ax2 + bx + c = 0 is satisfied by more than Hence, it is an equation.
two values of x, then a = b = c = 0.
or
In an identity in x coefficients of similar powers of x on
Linear Equation
the two sides are equal. An equation of the form
For example, If ax4 + bx3 + ex2 + dx + e ax + b = 0
= 5x4 -3x3 + 4x2 -7x -9 be an identity in x, then where a, b G R and a *0, is a linear equation.
i b
a = 5,fe = -3,c = 4,d = -7,e = -9. Eq. (i) has an unique root equal to —.
a
Thus, an identity in x satisfied by all values of x, where as
an equation in x is satisfied by some particular values of x.
106 Textbook of Algebra

(3X-1) =1-* A root of the quadratic Eq. (i) is a complex number a, such
I Example 4. Solve the equation - +
6 2 that aa2 + ba + c = 0. Recall that D = b2 - 4ac is the
Sol. We have, - + (3A-—^ = 1-- discriminant of the Eq. (i) and its roots are given by the
2 6 2 following formula.
x
-+-+-=]+-1
X X X ,
- b ± -J~D
or
2 2 2 6 x =------------ [Shridharacharya method]
3x 7 2a
or ------ ZS —

2--- 6
7
or x=-
9
Nature of Roots
1. If a, b, c E R and a 0, then
E Example 5. Solve the equation (a - 3)x + 5 = o+2. (i) If D < 0, then Eq. (i) has non-real complex roots.
Sol. Case I For a * 3 , this equation is linear, then (ii) If D > 0, then Eq. (i) has real and distinct roots,
(a-3)x = (a-3) namely
— b + -J~D — b — y/~D
(a-3) and then
*1 =-------- ------------ ’ X2 =-------------------

Case II For a = 3, 2a 2a
0x+5=3+2 axz+tx+c = a(x-xj(x-x2). -...(ii)
=> 5=5
(iii) If D = 0, then Eq. (i) has real and equal roots, then
Therefore, any real number is its solution.
b
Xi = x2 - ------ and then
2a
Quadratic Equations ax2 +bx +c = a(x -Xj )2.
An equation in which the highest power of the unknown ...(iii)
quantity is 2, is called a quadratic equation. To represent the quadratic ax2 + bx + c in form
Quadratic equations are of two types :
Eqs. (ii) or (iii), is to expand it into linear factors.

1. Purely Quadratic Equation (iv) If D > 0, then Eq. (i) has real roots.

A quadratic equation in which the term containing the (v) If D} and D2 be the discriminants of two
first degree of the unknown quantity is absent, is called a quadratic equations, then
purely quadratic equation. (a) If + D2 >0, then
i.e., ax2+c = 0, • atleast one of and D2 >0.
where a, c G C and a 0. • if Dj <0, then D2 >0 and if >0, then
D2 <0-
2. Adfected Quadratic Equation (b) If £>! + D2 <0, then
A quadratic equation in which it contains the terms of • atleast one of and D2 < 0.
first as well as second degrees of the unknown quantity, is • If <0, then Dz >0 and if >0, then
called an adfected (or complete) quadratic equation. £>2 <0.
i.e., ax + bx + c = 0, 2. If a, b, c G Q and D is a perfect square of a rational
where a, b, c G C and a 0, b £ 0. number, the roots are rational and in case it is not a
perfect square, the roots are irrational.
3. If a, b, c G R and p + iq is one root of Eq. (i) (q * 0),
Standard Quadratic Equation then the other must be the conjugate (p - iq) and
An equation of the form vice-versa (where, p,qE R and i = V~l).
ax2+bx+c=0 ...(i) 4. If a, b, c G Q and p + Jq is one root of Eq. (i), then the

where a,b,cECand a *0, is called a standard quadratic other must be the conjugate p-Jq and vice-versa
equation. (where, p is a rational and Jq is a surd).
The numbers a, b, c are called the coefficients of this
5. If a = 1 and b,cE I and the roots of Eq. (i) are rational
equation.
numbers, these roots must be integers.
Chap 02 Theory of Equations 107

6. Ifa4-b4-c=0 and a, b, c are rational, 1 is a root of the I Example 8. Show that if p,q,rands are real numbers
Eq. (i) and roots of the Eq. (i) are rational. and pr = 2(q+s), then atleast one of the equations
7. a2 4- b2 4- c2 - ab - be - ca = -1
x24-px4-q = 0and x2 4-1X4-S =0 has real roots.
2 Sol. Let Dj and D2 be the discriminants of the given equations
{(a-i>)2+(b-c)2+(c-a)2} x2 4- px 4- q = 0 and x2 4- rx 4- $ = 0, respectively.

= - {(a - b) (b-c) +(b- c) (c - a) 4- (c - a) (a - i)} Now, D{ 4- D2 = p2 - 4q 4- r2 - 4s = p2 4- r2 - 4(q 4- s)


= p2+r2-2pr [given, pr = 2{q 4- s)]
S Example 6. Find all values of the parameter a for
= (p - r)2 > 0 [•.• p and q are real]
which the quadratic equation
or Dj 4- D2 > 0
(O4-1)x2 4-2(04-1)x4-o-2 = 0 Hence, atleast one of the equations x2 4- px 4- q = 0 and
(i) has two distinct roots. x2 4- rx 4- s = 0 has real roots.
(ii) has no roots. I Example 9. If a,0 are the roots of the equation
(iii) has two equal roots.
(x - a) (x - b) = c, c 0. Find the roots of the equation
Sol. By the hypothesis, this equation is quadratic and therefore (x-a)(x-p)+c = 0.
at-l and the discriminant of this equation, Sol. Since, a, 0 are the roots of
D = 4(a4-1)2 - 4(a 4-l)(a-2) (x-a)(x-b) = c
= 4(a 4-1)(a4-l-a4-2) or (x - a)(x - b) - c =0,
= 12(a4-l) Then (x-a)(x- b)-c = (x-a)(x-0)
(i) For a > (-1), then D > 0, this equation has two distinct (x -a)(x - 0) + c = (x - a)(x - b)
roots. Hence, roots of(x -a)(x - 0) + c = Oare a, b.
(ii) For a < (- 1), then D < 0, this equation has no roots.
I Example 10. Find all roots of the equation
(iii) This equation cannot have two equal roots. Since,
D = 0 only for a = -1 and this contradicts the x4 +2x3-16x2 -22x + 7 = 0, if one root is 24- V3.
hypothesis. Sol. All coefficients are real, irrational roots will occur in
conjugate pairs.
I Example 7. Solve for x,
Hence, another root is 2 - ^3.
(54-2V6)*2"3 4-(5-2V6)x 2 “3=10.
Product of these roots = (x - 2 - -Ji) (x - 2 + -Ji)
Sol. v (5 + 2^6) (5 - 2^6) = 1
= (x-2)2-3=x2 -4x4-1.
1___
(5 - 2^6) = On dividing x4 + 2x3 - 16x2 - 22x 4- 7 by x2 - 4x 4-1, then
(5 4- 2^6)
the other quadratic factor is x2 4- 6x 4- 7.
(5 + 2^/6)x2-3 +(5-2>/6)x2"3 = 10
Then, the given equation reduce in the form
xz-3
(x2 — 4x4-l)(x2 + 6x4-7) = 0
reduces to = 10
^5 4- 2V6 ) x2 4- 6x 4- 7 = 0

Put (5 + 2a/6)x2"3 = t, then t + - = 10 Then,


t 2
t2 -1014-1 = 0 Hence, the other roots are 2 - -Ji, - 3 ± -Ji.

or
2 Relation between Roots
=> (5 4-2>/6)x2"3 = (5 ± 2a/6) = (5 + 2>/6)±1 and Coefficients
x 2“ - 3 = ± 1 1. Relation between roots and coefficients of
quadratic equation If roots of the equation
x2-3 = lorx2-3 = -l
ax2 4- bx 4- c = 0 (a * 0) be real and distinct and a < 0,
=> x2 = 4or xz = 2 -b + -J~D a -b-jD
Hence, x = ± 2, ±V2 then a = ------------- ,.3 =
P --
2a---------------- 2a

108 Textbook of Algebra

(i) Sum of roots (vii) a5 + p5 =(a2 + p2)(a3 + p3)-a2p2(a + 0)


Coefficient of x
= S = a +P = - —
a Coefficient ofx2
' b2 -2aP '_(P-3abc)y P_ by
a“ 2 a3 a2 a,
(ii) Product of roots k 7 k
„ o c Constant term _ ~(b5 -5ab3c +5a2bc2)
= P =ap = - =-----------------
a Coefficient of x2
(iii) Difference of roots (viii) a5 -p5 =(a2 + p2)(a3 -p3) +a2 p2(a -P)
^Discriminant
= D'=a-P = — ' b2 -2ac <5/D(D+3ac) 2
a Coefficient of x2 c
+
2. Formation of an equation with given roots k a
2
73 a k a 7
A quadratic equation whose roots are a and p, is _jD(b4 -3acb2 + 3a2c2)
given by (x - a) (x -p) =0 or x2 - (a + P)x +aP = 0
a5
i.e. x2 - (Sum of roots) x + Product of roots = 0
I Example 11. If one root of the equation
x2-Sx + P=Q. x2 - ix - (1 + I) = 0, (i = V-1) is 1 + i, find the other root.
3. Symmetric function of roots A function of a and
Sol. All coefficients of the given equation are not real, then
p is said to be symmetric function, if it remains other root £ 1 - i.
unchanged, when a and p are interchanged. Let other root be a, then sum of roots = i
For example, a3 +3a2 P + 3ap2 +p3 is a symmetric i.e. 1 + i + a = i => a = (-1)
function of a and p, whereas a3 ~p3 + 5aPis not a Hence, the other root is (-1).
symmetric function of a and p. In order to find the G Example 12. If one root of the equation
value of a symmetric function in terms of a + P, aP
and a - p and also in terms of a, b and c. x2 - V5 x -19 = 0 is-------- , then find the other root.
2
(i) a2 +P2 =(a + P)2 -2aP Sol. All coefficients of the given equation are not rational,
f 2
b b2 -2ac , i 9-75
-2 then other root ----------.
2
k a a2
Let other root be a, sum of roots = 5/5
(ii) a2-p2 =(a+0)(a-0) 9 + 75 r- -9 + ^5
/ => -------- + a = 5/5 => 01 =------------
( b} ID^
= ——
b^D 2 2
k aJ\ a 7 a2 Hence, other root is
(iii) a3 +p3 =(a+p)3 -3aP(a+p)
7 ,3 , \
b
3 7
c '_b b3 -3abc G Example 13. If the difference between the
-3 corresponding roots of the equations x2 + ox + b = 0
a k« k a < a3
7
and x2 + bx + o = 0(o^b) is the same, find the value
(iv) a3 -p3 =(a-P)3 +3ap(a -p)
of a+b.
r fp'3 + 3 -V lDy y[D(D+3ac) Sol. Let a, p be the roots of x2 + ax + b = 0 and y, 5 be the
a a J a3 roots of x2 + bx + a = 0, then given
k “ 7

(v) a4 +p4 =(a2 +p2)2 -2a2 p2: a-P=y-8


2 Ja2 - 4b _ ^b2 - 4a
b — 2ac b4 +2a2c2 -4acb2 va-p = —
-2 1 1 a
a2
\ M 7 ka a4
=> a2 - 4b = b2 - 4a
(vi) a4 ~P4 =(a2+p2)(a2-p2) => (a2 - b2) + 4(a - b) = 0 => (a - b)(a + b + 4) = 0
feVo (b2 -2ac) a-b*0
a4 a + b + 4= 0ora+h = -4.
Chap 02 Theory of Equations 109

I Example 14.If o + b + c = 0and a,b,c are rational. I Example 16. If a is a root of.4x2 + 2x -1 = 0. Prove
that 4a3 - 3a is the other root.
Prove that the roots of the equation
(b + c-o)x2 + (c + o-b)x+(o+b-c) = 0 Sol. Let other root is p,
, O 2 1 o1 1
are rational. then a + p = — = — - - or P =------ a (0
4 22 2
Sol. Given equation is
and so 4a2 + 2a - 1 = 0, because a is a root of
(b + c - a)x2 + (c + a - b)x + (a + b - c) = 0 •(i)
4x2 + 2x - 1 = 0.
v (b + c - a) + (c + a - b) + (a + b - c) = a + b + c = 0
Now, p = 4a3 -3a = a(4a2 -3)
x = 1 is a root of Eq. (i), let other root of Eq. (i) is a, then
= a(l -2a -3) [•.•4a2 + 2a -1=0]
_ , r a+b-c
Product of roots =----------- = -2a2 - 2a
b+c-a
—c — c = --(4a2)-2a
=> 1 xa =-------- [v a + b + c = 0] 2
-a - a
= --(1 -2a)-2a [•.•4a2 + 2a -1 = 0]
c - 2
a=- [rational]
a
Hence, both roots of Eq. (i) are rational.
.-l-a=P [from Eq. (i)]

Aliter Hence, 4a3 - 3 a is the other root.


Let b + c - a = A,c + a - b = B, a + b-c-C
Then, A +B+C=0 [v a + b + c = 0] ...(ii) I Example 17. If a,p are the roots of the equation
Now, Eq. (i) becomes X(x2 - x)+ x+ 5 = 0. If X] and X2 are two values of X
Ax2 + Bx + C - 0 ...(iii) a B 4
for which the roots a,p are related by — + — = -, find
Discriminant of Eq. (iii),
D = B2 - 4 AC X
the value of 7^ + 7^-.
= (-C - A)2 - 4AC [v A + B + C = 0] XX2 X,
= (C + A)2 -4AC So/. The given equation can be written as
Xx2 - (X - l)x + 5 = 0
= (C- A)2 = (2a - 2c)2
va, P are the roots of this equation.
= 4(a - c)2 = A perfect square
X-l n 5
a+P= and aP = —
Hence, roots of Eq. (i) are rational. X A
a P 4
I Example 15. If the roots of equation But, given —+—=-
p a 5
a(b-c)x2 +b(c-a)x + c(a-b) = 0
a2 +p2 4
be equal, prove that a,b,c are in HP.
aP 5
Sol. Given equation is
(X-l)2 10
a(b - c)x2 + b(c - d)x + c(a-b) = 0 ...(i) (a + p)2 - 2ap _ 4 4
=> =>
X2 X
Here, coefficient of x2 + coefficient of x + constant term = 0 ap " 5 2 5
i.e., a(b - c) + b(c - a) + c(a - b) = 0
x
Then, 1 is a root of Eq. (i). (X - I)2 - IPX _ 4
=> X2 - 12X + 1 = 4X
Since, its roots are equal. 5X 5
Therefore, its other root will be also equal to 1. X2 - 16X + 1 = 0
, c(a-b) It is a quadratic in X, let roots be Xj and X2, then
Then, product of roots = 1x1 =----------
a(b - c) X] + X2 = 16 and XjX2 — 1
=> ab - ac = ca - be X]
X] X22 _ X2 +X2 _ (X, + X2)2 - 2XtX2
X
T± + — =
, 2ac X2 Xj
^-1 XjX2 XjX2
b =-----
a+c (16)2 -2(1)
= 254
Hence, a, b and c are in HP. 1
no Textbook of Algebra

6 Example 18. If a, 0 are the roots of the equation = (a + p)(a-P)2{(a +p)2 -aP}
x2 - px + q = 0, find the quadratic equation the roots = p(p2 -4q)(p2 -q)
of which are (a2 -p2)(a5 -p5) and a°02 + a2 p\
and a3p2+a2p3 = a2p2(a+p)=p<?2
Sol. Since, a, P are the roots of x2 - px + q = 0.
S = Sum of roots = p(p2 - 4g) (p2 - q) + pq2
a + P = p, ap = q
= p(p4 -5pzg +5g2)
=> a - p = 7(P2 -«<7) P = Product of roots = p2q2(p2 - 4g) (p2 - g)
Now, (a2 - p2)(a3 - P3) Required equation is x2 - Sx + P = 0
= (a + P)(a - P)(a - P)(a2 +ap + p2) i.e. x2 - p(p4 - 5p2g + 5g2)x + p2g2(p2 - 4g)(p2 - g) = 0

|5| Exercise for Session 1


1. If (a2 -1)x2 + (a -1)x + a2 -4a + 3 = 0 be an identity inx, then the value of a is/are
(a) —1 (b) 1 (c)3 (d)-1,1,3
2. The roots of the equation x
x 2 + 2V3x + 3 = 0 are
(a) real and unequal (b) rational and equal
(c) irrational and equal (d) irrational and unequal
3. Wa,b,CG Q.then roots of the equation (b + c -2a)x2 + (c + a -2b)x + (a + b -2c) = 0are
(a) rational (b) non-real (c) irrational (d) equal
4. If P(x) = ax2 + bx + cand Q(x) = -ax2 + dx + c, whereac #0,thenP(x)Q(x) = 0has atleast
(a) four real roots (b) two real roots
(c) four imaginary roots (d) None of these
5. If roots of the equation (q -r)x2 + (r -p)x + (p -q) = 0 are equal, then p,q,r are in
(a)AP (b)GP (c)HP (d)AGP
6. If one root of the quadratic equation ix2 - 2 (/ + 1) x + (2 - i) = 0, / = is 2 - /, the other root is
(a)-/ (b)/ (c)2+/ (d 2-i
7. If the difference of the roots of x2 - Ax + 8 = 0 be 2, the value of X is
(a) ±2 (b) ± 4 (c) ± 6 (d)±8
8. If 3p2 = 5p + 2and3q2 =5q + 2 where p*q,pq is equal to
2
(a)^ 2
(&)-■= 3
(c)^ (d)-|
O u z

9. If a, p are the roots of the quadratic equation x2 + bx - c = 0, the equation whose roots are b and c, is
(a) x2 + ax - P = 0 (b) x2 - [(a + p) + aP]x - ap (a + P) = 0
(c)x2 + [(a + P)+ aPJx + aP(a+ p) = 0 (d)x2 + [(a + P) + aP]x - ap(a+ P) = 0

10. Let p, q g {12,3,4}. The number of equations of the form px2 + qx + 1 = 0 having real roots, is
(a) 15 (b)9 (c)8 (d) 7
11. If a and Pare the roots of the equation ax2 + bx + c = 0(a *0,a,b,c being different), then
(1+ a+ a2)(1 + P + P2)is equal to
(a) zero (b) positive (c) negative (d) None of these

/
Session 2
Transformation of Quadratic Equations, Condition
for Common Roots
Transformation of S Example 19. If a,p be the roots of the equation
Quadratic Equations x2 -px + q = 0, then find the equation whose roots are
Let a, P be the roots of the equation ax2 +bx + c = 0, then —and—5—-
the equation
p-a P-P
(i) whose roots are a + k, P + k, is Sol. Let -q— = x => a = p - q—
p-a x
a(x-fc)2 + b(x-k) + c=0 [replace x by (x -k)]
So, we replacing x by p - — in the given equation, we get
(ii) whose roots are a - k, P - k, is x
/ \2 / \
a(x + k)2 +b(x + k)+c = 0 [replace x by (x + k)] -pp-— +<?=0
I Q )
X k xJ
(iii) whose roots are ak, p/c, is
92 2P<? p2 + M+q = <1
^+l^-iPl-
ax2 + kbx +k2c =0 replace x by f j- X‘
x X x

(iv) whose roots are —, is q-!H + ^o


k k X X
2 2 2
ak2 x2 +bkx + c=0 [replace x by xk] or qx - pqx + q =0 or x - px + q = 0
(v) whose roots are -a, - p, is is the required equation whose roots are —-— and
p-a P”P
ax2 - bx + c =0 [replace x by (-x)]

(vi) whose roots are —, is I Example 20. If a and p are the roots of
ox2 + bx + c = 0, then find the roots of the equation
ex2 +bx +a =0 1
replace x kby 1—
1 ox2 -bx(x-l) + c(x-l)2 =0.
Ix
(vii) whose roots are - Z, - , is Sol. v ax2 - bx(x - 1) + c(x - I)2 = 0 •••(*)
f \2 / \
x X
a -b + c =0
<X-1J kx-l,
ex2 - bx +a =0 i
replace x kby I —1 2
I x I X | X
k k or a I +b +c=0
(viii) whose roots are —, -, is II - X J J-x
Now, a, P are the roots of ax2 + bx + c = 0.
ex22 + kbx + k2a =0 replace x by —
Then, a = - X - and p = ——-
WJ 1-x 1-x
(ix) whose roots are pa + q, pP + q, is
2 / \ a j P
'x-q ' 'x-q^ x =------ and x = - ----
a + b x-q + c = 0 replace x by a+1 P+1
< p > k P > < P Hence, ———, ■ are the roots of the Eq. (i).
a+1 p+1
(x) whose roots are a", P", n e N, is
a(x1/n)2 +b(xl,n) + c=0 [replace x by (x1/n)] I Example 21. If a,P be the roots of the equation
(xi) whose roots are a1 ln, P1 ln, n e N is
zb-a?3 r1-P
3x2 + 2x +1 = 0, then find value of +
a(x")2 + b(x") + c=0 [replace x by (xn)]
J+a k1+P>
l-a i-x
Sol. Let -------- = X => a =------
1+a 1+x
112 Textbook of Algebra

1—x I Example 22. For what values of m, the equation


So, replacing x by------- in the given equation, we get
1+x x2 +2(m -1)x + m + 5 = 0 has (meR)
2
'l-x^ 1 - x^
3 + 2l + 1=0 => x2 -2x + 3 = 0 (i) roots are equal in magnitude but opposite in
J + X, 1 + Xj
sign?
, . , , 1 -q .1-p
It is clear that------- and------ are the roots of Eq. (i). (ii) roots are reciprocals to each other?
1+q I+P (iii) roots are opposite in sign?
fl-q 5 , fi-PY 2 -(ii) (iv) both roots are positive?
J+q, li + P; (v) both roots are negative?
l-aVl-Pk 3 (vi) atleast one root is positive?
and -(iii)
1 + a J 1^1 + P J (vii) atleast one root is negative?
3 Sol. Here, a = 1, b = 2(m - 1) and c = m + 5
'1 -a
3+(lsPt i-q , i-P
I -3 D = b2 - 4ac = 4(m - I)2 - 4(m + 5)
j + qJ II + pj 1+q 1+P
= 4(m2 - 3m - 4)
1 -a Y1-P
1 +a
f1 ~a
1 + P J^l + a 1 + p>
= 23 - 3-3-2 = 8 - 18= — 10
D = 4(m - 4)(m + l)and here a = 1 > 0
(i) b = 0 and D > 0
=> 2(m - 1) = 0 and 4(m - 4) (m + 1) > 0
Roots Under Special Cases => m = 1 and m e(-«>,-1) u(4, oo)
Consider the quadratic equation ax2 + bx + c = 0 ...(i)
mG $ [null set]
where a, b, c E R and a 0. Then, the following hold good : (ii) a = c and D > 0
(i) If roots of Eq. (i) are equal in magnitude but opposite in => 1 = m + 5 and 4(m - 4)(m + 1) > 0
sign, then sum of roots is zero as well as D > 0, i.e. b = 0 => m = - 4 and m G (-oo, -1] u [4, oo)
and D > 0.
.’. m=-4
(ii) If roots of Eq. (i) are reciprocal to each other, then product (iii) a > 0, c < 0 and D > 0
of roots is 1 as well as D > 0 i.e., a = c and D > 0.
=> 1 > 0, m + 5 < 0 and 4(m - 4) (m + 1) > 0
(iii) If roots of Eq. (i) are of opposite signs, then product of
=> m < - 5 and m G(-oo, -1) u(4, oo)
roots < 0 as well as D > 0 i.e., a > 0, c < 0 and D > 0 or
me (-00,-5)
a < 0, c > 0 and D > 0.
(iv) If both roots of Eq. (i) are positive, then sum and product of (iv) a>O,b < 0, c >0 and D > 0
roots > 0 as well as D > 0 i.e., a > 0, b <0, c > 0 and D > 0 or => 1 > 0,2(m - 1) < 0, m + 5 > 0
a <0,b >0,c <0 and D >0. and 4(m - 4)(m + 1) > 0
(v) If both roots of Eq. (i) are negative, then sum of roots < 0, m < 1, m > -5 and m G (-°°, -1] u [4, °°)
product of roots > 0 as well as D > 0 i.e., a >0, t >0, c > 0 me (-5,-1]
and D>0ora<0, fc<0, c<0 and D > 0.
(v) a > 0, b > 0, c > 0 and D > 0
(vi) If atleast one root of Eq. (i) is positive, then either one root
=> 1 >0,2(m -l)>0,m + 5>0
is positive or both roots are positive i.e., point (iii) U (iv).
and 4(m - 4)(m + 1) > 0
(vii) If atleast one root of Eq. (i) is negative, then either one root
is negative or both roots are negative i.e., point (iii) u (v). => m > 1, m > - 5 and m G (-oo, -1] u [4,00)
(viii) If greater root in magnitude of Eq. (i) is positive, then m G [4, oo)
sign of b = sign of c sign of a. (vi) Either one root is positive or both roots are
(ix) If greater root in magnitude of Eq. (i) is negative, then positive
sign of a = sign of b * sign of c. i.e., (c) u (d)

(x) If both roots of Eq. (i) are zero, then b = c = 0. => m G (-oo, -5) u(-5, -1]
( b\ (vii) Either one root is negative or both roots are
(xi) If roots of Eq. (i) are 0 and — , then c = 0.
I aj negative
Q i.e., (c)u(e)
(xii) If roots of Eq. (i) are 1 and —, then a + b + c - 0.
a m e (-oo.-S) u [4, oo)
Chap 02 Theory of Equations 113

Condition for Common Roots I Example 23. Find the value of X, so that the
equations x2 - x -12 = 0 and Xx2 + 10x + 3 = 0 may
1. Only One Root is Common have one root in common. Also, find the common root.
Sol. •: x2 -x-12 = 0
Consider two quadratic equations
(x-4)(x + 3) = 0
ax2 + bx + c = 0 anda'x2 + b' x + c' =0
x = 4,-3
[where a, a 0 and ab' -a' b*0] If x = 4 is a common root, then
Let a be a common root, then X(4)2 + 10(4)+ 3 = 0
a a2 +ha + c = 0anda'a2 + b'a + c' =0.
On solving these two equations by cross-multiplication, 16
we have and if x = - 3 is a common root, then
a2 a 1 X(-3)2 + 10(-3) + 3 = 0
bc'-b'c ca'-c'a ab' -a'b X=3
43
From first two relations, we get Hence, for X =-----, common root is x = 4
16
be' -b' c
a =----------- ...(i) and for X = 3, common root is x = - 3.
ca' -c' a
and from last two relations, we get I Example 24. If equations ox2+bx + c = 0, (where
ca' -c a a,b,ceR and o^O) and x2 + 2x +3 = 0 have a common
a =—~-- -(ii)
ab' -a'b root, then show that o: b: c = 1:2:3.
From Eqs. (i) and (ii), we get Sol. Given equations are
'be' -b'c ca' -c' a ax,22 + bx + c = 0 ...(i)
ca -c a ab'-a'b and x2 + 2x + 3 = 0 -(ii)
2
(ab' - a' b) (be' - b' c)=(ca' - c' a) Clearly, roots of Eq. (ii) are imaginary, since Eqs. (i) and (ii)
2 have a common root. Therefore, common root must be
a b b c c a imaginary and hence both roots will be common.
or x [remember]
a' b' b' c c a Therefore, Eqs. (i) and (ii) are identical.
This is the required condition for one root of two a b c
- = - = - or a:b:c = l:2:3
quadratic equations to be common. 1 2 3

2. Both Roots are Common I Example 25. If a,b,c are in GP, show that the
equations ox2 + 2bx + c = 0 and dx2 + 2ex + f = 0 have
Let a, P be the common roots of the equations ra b c
ax2 + bx + c = 0 and a' x2 +b' x +c' = 0, then a common root, if-,-,- are in HP.
def
D b b'
b' aa b Sol. Given equations are
...(iii)
a a
a' a'
a b' ax2 +2bx + c =0 ,.(i)
n c c a c
and ap = - = - -(iv) and dx2 +2ex + f =0 (n)
a a a' c'
Since, a, b, c are in GP.
a _b_ c
From Eqs. (iii) and (iv), we get
a' b' c b2 =ac or b = ac
This is the required condition for both roots of two From Eq. (i), ax2 + =0
quadratic equations to be identical. Vc
or (Va x + Vc)2 =0 or X - ---- y=
Remark va
To find the common root between the two equations, make the v Given Eqs. (i) and (ii) have a common root.
same coefficient of x2 in both equations and then subtract of the vC
two equations. Hence, x = —f= also satisfied Eq. (ii), then
va
114 Textbook of Algebra

d -c -2c^ + / = 0 d + f = 2e
-
or
' a ‘ a a c b
def
=>
d -2c + f - on L are in AP.
a b c
a Vac c
a b c
■d 2e f n Hence, — are in HP.
or [v b = Vac ] def
a b c

g Exercise for Session 2


1. If a and p are the roots of the equation 2x2 - 3x + 4 = 0, then the equation whose roots are a2 and p2, is
(a)4x2 + 7x + 16= 0 (b) 4x2 + 7x + 6 = 0 (c) 4x2 + 7x + 1= 0 (d) 4x2 - 7x + 16 = 0

11.
2. If a, p are the roots of x2 - 3x + 1 = 0, then the equation whose roots are ----1 ,------- L is
l<x-2 P-2J
(a)x2 + x - 1=0 (b) x2 + x + 1= 0 (c) x2 - x - 1 = 0 (d) None of these

3. The equation formed by decreasing each root of ax2 + bx + c = 0by 1 is2x2 + 8x + 2 =0, then
(a)a = -b (b)b=-c (c)c = -a (d)b=a + c
x2 — bx m —1
4. If the roots of equation ---------- =-------- are equal but opposite in sign, then the value of m will be
ax - c m +1
(a)^
a+b a+b
(c)i4
a-b
(d)^
b -a

5. If x2 + px + q = 0 is the quadratic equation whose roots are a - 2 and b - 2, where a and b are the roots of
x2 -3x + 1 = 0, then
(a) p = 1 q = 5 (b) p = 1 q = - 5 (c) p = -1 q = 1 (d) None of these
6. If both roots of the equation x2 - (m - 3) x + m = 0 (m e R) are positive, then
(a) m e (3, «>) (b) m g(-*>, 1] (c)n? e [9,«) (d)me(1 3)
7. If the equation (1 + m)x2-2(1 + 3m)x + (1 + 8m) = 0, where m e R ~ {-1}, has atleast one root is negative, then
(b)me^,« (d)meR

8. If both the roots of X(6x2 + 3) + rx + 2x2 -1 = 0and6X(2x2 +1)+ px +4x2 -2 = 0 are common, then 2r - pis
equal to
(a)-1 (b)0 ‘ (c)1 (d)2
a3 + b3 + c
9. If ax2 + bx + c = 0 and bx2 + cx + a =0 have a common root a *0,then - is equal to
abc
(a)1 (b)2 (c)3 (d) None of these
70. If a (p + q )2 + 2bpq + c = 0 and a (p + r)2 + 2bpr + c = 0, then qr is equal to

(a)p2 + - (b)p2 + - (c)p2 + £ (d) p2 + -


a c b a
Session 3
Quadratic Expression, Wavy Curve Method, Condition
for Resolution into Linear Factors, Location of Roots

Quadratic Expression 4. Intersection with axes


An expression of the form ax2 + bx + c, where a, b, c E R (i) Intersection with X-axis
and a *0 is called a quadratic expression in x. So, in For X-axis, y =0.
general quadratic expression is represented by -b±y/D
ax2 + bx +c =0 => x =-----------
/(x) = ax2 + bx + c or y = ax2 + bx + c. 2a
For D > 0, parabola cuts X-axis in two real and
Graph of a Quadratic Expression distinct points
a < 0, D > 0
We have, y - ax2 +bx + c = fix'), [a ^0]

y=a -T
2a
D
4a2 ------- X-axis
X-axis

a > 0, D > 0
2
D\ b}
or y +— =a x + — For D = 0, parabola touches X-axis in one point
4a J 1 2a)
b
i.e., x - ------ .
Now, let y +—=Y and x + — = X 2a
4a 2a a<0, D = 0
2
Y = aX X-axis

x2^-
a X-axis
1. The shape of the curve y = /(x) is parabolic. a >0, D = 0

2. The axis of parabola isX=0orx + — = 0 For D < 0, parabola does not cut X-axis i.e.,
L imaginary values of x.
or x = - — i.e. parallel to Y-axis. a < 0, D < 0
•X-axis

3. (i) If - > 0 => a > 0, the parabola open upwards.


a
-X-axis
a > 0, 0 < 0

(ii) Intersection with Y-axis


For Y-axis, x = 0.
(ii) If - < 0 => a < 0, the parabola open downwards.
a /. y=c
5. Greatest and least values ofy(x)
2 1
Vertex of the parabola X = - Y is
a
X=0,Y=0
116 Textbook of Algebra

b n D n 3. a > 0 and D = 0. So, /(x) > 0 for all x E R,


x H---- = 0,y -I----- =0
2a 4a i.e. f(x) is positive for all real values of x except at
b D vertex, where /(x) = 0.
or x=~ — ,.V =-----
2a 4a
2 ( b D
Hence, vertex of y = ax + bx + c is----- ,
V 2a 4a
I . I Vertex X-axis

a >0 4. a < 0 and D = 0. So, f(x) < 0 for all x G R,


a <0
i.e. f(x) is negative for all real values of x except at
Vertex vertex, where f(x) = 0.
b_ X-axis
For a > 0, f(x) has least value at x = -
2a
_D_
This least value is given by f | | =-
I 2a J 4a
D
or y least ~ ■
4a 5. a > 0 and D > 0.
( D A
Range of y - ax 2 + bx + c is Let /(x) = 0 have two real roots a and 0 (a < 0), then
I 4a ) /(x) >0 for all x c(-«>,a)u (0, °°) and/(x) <0 for
b_
For a < 0, /(x) has greatest value at x = - all x G (a, 0).
2a
D_
This greatest value is given by f\-----
2a j 4a
D
or y greatest ~ a >0
4a
D X-a
Range of y = ax 2 + bx + c is ------
4a
6. a < 0 and D > 0

Sign of Quadratic Expression Let /(x) = 0 have two real roots a and 0 (a < 0),
then /(x) < 0 for all x G (- oo, a) U (0, °°)
Let /(x) = ax2 + bx + c or y = ax2 + bx + c,
and /(x) > 0 for all x G (a, 0).
where a, b, c G R and a * 0, for some values of x, /(x) may
be positive, negative or zero. This gives the following X-axis
cases :
1. a > 0 and D < 0.
So, /(x) > 0 for all x G R,
i.e. /(x) is positive for all real values of x.
Wavy Curve Method
a >0
(Generalised Method of Intervals)
Wave Curve Method is used for solving inequalities of the
X-axis
form
2. a < 0 and D < 0. So, /(x) < 0 for all x G R, _r(x-a1)kl(x-a2)k2...(x-am)
/(x) — >0
i.e. /(x) is negative for all real values of x. ’ (x-bl)p'(x-b2)p2...(x-bn)Pn
I
------------- - --------- ► X-axis
(<0, >0or<0),'
a <0 where, kitk2,...,km, pi,p2»-->Pn are natural numbers and
I
such that a( b, where i = 1,2 m and j = 1,2,..., n.
Chap 02 Theory of Equations 117

We use the following methods: Important Results


1. Solve(x -ax )fcl(x -a2)k2 ... (x - a m ) = 0 and 1. The point where denominator is zero or function approaches
infinity, will never be included in the answer.
(x - bi)pl(x - a2)P2 ...(x - bn)Pn =0, then we get. 2. Forx2 <a2or|x| <a « -a<x <a
x = alta2..... am, bu b2,..., bn [critical points] i.e., xe(-aa)
2. AssumeOj <a2 <...<am <b} <b2 <...<bn 3. ForO < x22 < a2 orO <|x| < a

Plot them on the real line. Arrange inked (black) « -a<x<a~{0}


i.e., xe(-aa)~{0]
circles (•) and un-inked (white) circles (O), such
4. Forx2 > a2 or|x| > a <=> x <-aorx>a
that
i.e., x e(-oo,-a] u[a,«>)
al a2 ••• am bi b2 ... bn
5. Forx2 >a2or|x| >a <=> x<-aorx>a
If/(x)>0 O O... O O O... O
i.e., x e(- oo, - a) u(a ~)
M<q O O... O O O... O
6. Fora2 < x2 < b2 ora <|x| < b
f(x)>0 O O... O
<=> a<x <bor-b<x <-a
M<q O O... O i.e., x e [- b, - a] u [a b]
3. Obviously, bn is the greatest root. If in all brackets 7. Fora2 <x2 < b2ora<|x| < b
before x positive sign and expression has also <=> a<x <bor-b<x <-a
positive sign, then wave start from right to left, i.e., x e [- b, - a) u (a b]
beginning above the number line, i.e. 8. For a2 < x2 < b2 or a < |x| < b
, (x-a,)t|(x-a2)t;...(x-a,)* m
—,then <=> a<x <b or - b<x a
(x-(,1)f(x-(,2r...(x-6„)'’" i.e.. xe(-b,-a]u[ab)
9. For a2 < x2 < b2 or a < | x| < b

« a<x <bor-b<x <-a


+
i.e., x e(-b, - a) u(a b)

bn 10. For (x - a)(x - b) < 0 and a < b, then a < x < b


i.e., xe(ab)
and if in all brackets before x positive sign and 11. If (x - a) (x - b) <0 anda< b,
expression has negative sign, then wave start from then a < x < b, x e[a b]
right to left, beginning below the number line, i.e. 12. If (x - a)(x - b) >0 and a < b, then x < aorx > b
(x-q,)t'(x-a2)t‘...(x-a„)t“ i.e., xe(-=»a)u(b,°°)
, then
(x-h1f‘(x-l>2)'’!...(x-l>,)f'" 13. If (x — a) (x — b) >0 anda< b,
then x < aorx > b
t>»
i.e., x g (- °°, a] u [b, °o)

I Example 26. Solve the inequality


(x + 3)(3x-2)5(7 -x)3(5x + 8)2 >0.
4. If roots occur even times, then sign remain same Sol. We have, (x + 3)(3x - 2)5(7 - x)3(5x + 8)2 > 0
from right to left side of the roots and if roots => - (x + 3)(3x - 2)5(x - 7)3(5x + 8)2 > 0
occur odd times, then sign will change from right to (x + 3) (3x - 2)s (x - 7)3(5x + 8)2 < 0
left through the roots of [take before x, + ve sign in all brackets]
x = a{,a2,...,am,b1,b2,...,bn.
-3 4- j f 4-
5. The solution of /(x) > 0 or /(x) > 0 is the union
__8 2 7
of all intervals in which we have put the plus sign 5 3
and the solution of /(x) <0 or f(x) <0 is the
union of all intervals in which we have put the 8 2
The critical points are (- 3), - - , 7.
minus sign. 5/3
2 8
Hence, x e (-«>,-3] U -,7 u
5
118 Textbook of Algebra

+\ < +
I Example 27. Solve the inequality “T T“
971 "3 2
( 1
(X - 2)10000 (x + 1)253 (x + 8)4 1 1 2
I__ 2 >0
Hence, x G (— 2, — 1) kJ — —
V 3 2
x 500(x — 3)75 (x + 2)93
1
97! G Example 30. For xgR, prove that the given
(x- 2)10000 (x +1)253 x — | U + 8)4
____2 x2 + 34 x - 71
Sol. We have, >0 expression —r----------- cannot lie between 5 and 9.
x 500(x - 3)75(x + 2)93 x2 + 2x-7
^2z +34X-71
x
The critical points are (- 8), (- 2), (- 1), 0, 2,3. Sol. Let — --------- =y
x +2x-7
[vx*-2,0,3]

4 + v +N 2 { 4“

-8 0 1 3 x2 (y - 1) + (2y - 34 )x + 71 - 7y = 0
2
For real values of x, discriminant > 0
( 11
Hence, x G (- «>,',-8] kJ [-8,-2) kJ [-1,0) kJ 0,- kj(3, °°) (2y - 34)2 - 4(y - 1)(71 - 7y) > 0
I 2J
1 => 8y2 - 112y + 360 > 0
or xg(-°o -2) kJ [-1,0) kJ 0, — kJ (3, oo)
2 => y2 - 14y + 45 > 0

(x — 3)(x + 2)(x + 6) (y-9)(y-5)>0


S Example 28. Let f(x) =
(x + 1)(x —5) => yG(- °°, 5] kJ [9, °o)
Find intervals, where /(x) is positive or negative. Hence, y can never lie between 5 and 9.

Sol. We have, f(x)~--------- -------- ------- -


(x + l)(x-5)
G Example 31. For what va ues of the parameter k in
x2 + kx + 1
The critical points are (- 6), (- 2), (- 1), 3,5 the inequality < 3, satisfied for all real
x2 + x + 1
values of x ?
x2 + kx + 1
Sol. We have, <3
x2 + x + 1
For f(x) > 0, V x e (- 6, - 2) kJ (- 1,3) kJ (5, ■»)
„ x2 + kx + 1 „
For /(x) < 0, V x G (- °°, - 6) kJ (- 2, - 1) kJ (3,5) - 3 < —----------- < 3
x +x+1
I Example 29. Find the set of all x for which 1
2x __ i Since, x2 + x + 1 = x + — |+3>o
2 1 4
(2x2 + 5x + 2) (x + 1)'
- 3(x2 + x + 1) < x2 + kx + 1 < 3(x2 + x + 1)
2x 1
Sol. We have, 4x2 + (k + 3)x + 4 > 0 ...(0
(2x2 + 5x + 2) (x + 1)
and 2x2 - (k - 3)x + 2 > 0 ...(ii)
2x 1
=> __________ >0 4 > 0 and 2 > 0
(x+2)(2x + l) (x + 1)
The inequality (i) will be valid, if
(2x2 +2x)-(2x2 +5x + 2) (k + 3)2 -4-4-4 <0=> (k + 3)2 <64
=> >0
(x + 2)(x + l)(2x + 1) or -8<k+3<8
or - 11 < k < 5 •(iii)
______________ ______ >0
(x + 2)(x + l)(2x + l) and the inequality (ii) will be valid, if
(k-3)2 -4-2-2<0 or (k-3)2 <16
______<3* + 2>. ...-<0
or or -4 <k-3<4
(x+2)(x + l)(2x+l)
or - 1 <k <7 „.(iv)
( 21 I 1 The conditions (iii) and (iv) will hold simultaneously, if
The critical points are (- 2), (-1), I - - LI
2
—1<k <5
Chap 02 Theory of Equations 119

Condition for Resolution I Example 34. Find the linear factors of


x2 - 5xy+ 4y2 + x + 2y -2
into Linear Factors Sol. Given expression is
The quadratic function x2 - Sxy + 4y2 + x + 2y - 2
f(x, y) = ax2 + 2hxy + by2 + 2gx + 2fy +c Its corresponding equation is
may be resolved into two linear factors, iff x2 - 5xy + 4y2 + x + 2y - 2 = 0
A = abc + 2fgh - af2 - bg2 -ch2 =0 or x2 - x(5y - 1) + 4y2 + 2y - 2 = 0
_ (5y - 1) ±-J(5y - I)2 - 4-1-(4y2 + 2y - 2)
ahg
______ 2
i.e., h b f =0 _(5y-l)±7(9y2-18y + 9)
S f c 2____
_(5y-l)±V(3y-3)2
I Example 32. Find the value of m for which the 2
expression 12x2 -10xy + 2y2 + 1lx-5y + m can be
J.^(fr-.?) = 4y-2.y+l
resolved into two rational linear factors.
Sol. Comparing the given expression with :. The required linear factors are(x - 4y + 2) and(x - y - 1).
ax2 + 2hxy + by2 + 2gx + 2fy + c , we have
a = 12,h = -5,b = 2,g=^,f = \- -53L c = m
2)
Location of Roots
The given expression will have two linear factors, if and [Interval in which Roots Lie]
only if
Let f(x) = ax2 +bx +c, a,b,ce R,a*0 and a, p be the
abc + 2fgh - af2 - bg2 - ch2 = 0
2
roots of f(x) =0. Suppose k, klt k2 e R and kx <k2. Then,
5
or (12)(2)(m) + 2[ - - lT|(-5)-(12)f- the following hold good :
I 2 <2J 2
2
I -(m)(-5)2 =0 1. Conditions for Number k
275 121 (If both the roots of /(x) =0 are less than k)
=> 24 m +------ 75 ---- 25 m = 0 or m = 2
2 2
I i a><> ; I 4a.
8 Example 33. If the expression
ax2 + by2 + cz2 + 2ayz + 2bzx + 2cxy can be resolved
into two rational factors, prove that
a3+b3 + c3 = 3abc.
\Wr X-axis
♦■X-axis

/ b_ _ D_\
Sol. Given expression is \ 2a ’ 4a/

ax2 + by2 + cz2 + 2ayz + 2bzx + 2cxy (0


' / \2 / 2 (i) D > 0 (roots may be equal)
x + by- I y i , i Xx i n x y
= z2 a + c + 2a - | + 2t| - | + 2c| - (ii) fl/(fc)>0
z lZ z z z
= z2 [aX2 +bY2 +c+2aY + 2bX + 2cXY] (iii) k > - —, where a < |3.
2a
where, — = X and — = Y
z z I Example 35. Find the values of m, for which both
Expression (i) will have two rational linear factors in x, y roots of equation x2 - mx +1 = 0 are less than unity.
and z, if expression
Sol. Let f(x) = x2 - mx + 1, as both roots of f(x) = 0 are less
aX2 + bY2 4- 2cXY + 2bX +2aY + c will have two linear
than 1, we can take D > 0, qf(l) > 0 and----- <1.
factors, if 2a
abc + 2abc - aa2 - bb2 - cc2 = 0

or a3 + b2 +c3 = 3abc
120 Textbook of Algebra

(i) Consider D > 0 (- m)2 - 4 • 1 • 1 > 0 11Y A


=> m----- (m - 1) > 0
=> (m + 2)(m-2)>0 9 J
=> m G (-00, - 2] 0(2, 00)
=> mG(- x I—
/ “, l)ul 11 , 00 (ii)
(ii) Consider af (1) > 0 1(1 - m + 1) > 0 9
(iii) Consider f- — > 3
=> m-2<0=^m<2
I 2a
=> me (- «,2) (ii)
(iii) Consider f- — < 11 6m n
— >3
I 2a J 2
=> m>1
— < 1 => m < 2 m e (1, oo) ...(iii)
2
Hence, the values of m satisfying Eqs. (i), (ii) and (iii)
=> m G (- oo, 2) -(iii)
Hence, the values of m satisfying Eqs. (i), (ii) and (iii) at the same time are m G | —, °° |.
I 9 J
at the same time are m G (— °°, — 2].

2. Conditions for a Number k 3. Conditions for a Number k


If k lies between the roots of /(x) = 0
If both the roots of /(x) = 0 are greater than k

II 'I a>0 ''I


/ I \ 2a 4a/ I
\ 8>0 /I U-&) ,

\ / X-axis / \ X-axis
/ a<0 \
* \ l . .x^s '(*);/ / 23 \ \
( 2a ’ 4a) I I
'a<0'
\ 2a 4a/ I ! J I
(i)D>0 (ii) af(k) < 0, where a < P
(i) D >0 (roots may be equal)
(ii) a/(k)>0 6 Example 37. Find all values of p, so that 6 lies
(iii) k < - —, where a < 0. between roots of the equation x2 + 2(p - 3)x + 9 = 0.
2a
Sol. Let f(x)= x2 + 2(p - 3)x + 9, as 6 lies between the roots
I Example 36. For what values of m eR, both roots of of /(x) = 0, we can take D > 0 and qf(6) < 0

the equation x2 -6mx+9m2 - 2m - 2 = 0 exceed 3? (i) Consider D > 0

Sol. Let f(x) = x2 - 6mx + 9m2 - 2m + 2


As both roots of f(x) = 0 are greater than 3, we can take
{2(p -3)}2 -4-1-9 >0
D > 0, af(3) > 0 and- — > 3.
2a (p - 3)2 - 9 > 0
(i) Consider D > 0
=> p(p-6)>0
(-6m)2 - 4-l(9m2 -2m + 2) > 0 => 8m-8>0
=> p e (- co, o) u (6, °°) .(i)
m > 1 or me [1,~) -(i) (ii) Consider a /(6)<0
1-{36 + 12(p - 3) + 9} < 0
(ii) Consider a/(3) > 0 3
=> 12p + 9 < 0 => p + — < 0
' l-(9 - 18m + 9m2 -2m + 2)>0
3^
+ > <+ - °°. ~ - ■(ii)
1 7i/9 47
Hence, the values of p satisfying Eqs. (i) and (ii) at the
=> 9m2 -20m + 11 >0
3
same time are p G — oo,
(9m - ll)(m - 1) >0 4
Chap 02 Theory of Equations 121

4. Conditions for Numbers and k2 I Example 39. Find all values of a for which the
equation 4x2 -2x + a = 0 has two roots lie in the
If exactly one root of /(x) = 0 lies in the interval (/q, k2)
interval (-1,1).

.1 - I
i M . 7-\
X-axis
Sol. Let f(x) = 4x2 - 2x + a as both roots of the equation,
f(x) = 0 are lie between (- 1,1), we can take D > 0,
af(- 1) > 0> a/(J) > 0 and - 1 < — < 1.
4
/q a \ k2 /pX-axis I j
(i) Consider D > 0

(i) D >0 (-2)2 - 4-4-a > 0 => a<-


4
—(0
(ii) fUi) f(k2) < 0, where a < P-
(ii) Consider a f(- 1) > 0
I Example 38. Find the values of m, for which exactly 4(4 + 2 + a)>0
one root of the equation x2 - 2mx + m2 -1 = 0 lies in => a > - 6 => a G (- 6, °°) -(ii)
the interval (-2,4). (iii) Consider a J(l) > 0
Sol. Let f(x) = x2 - 2mx + m2 - 1, as exactly one root of 4(4-2 + a)>0 => a>-2
f(x) = 0 lies in the interval (-2, 4), we can take D > 0 and => ae(-2,co) ...(iii)
/(-2)/(4)<0.
Hence, the values of a satisfying Eqs. (i), (ii) and (iii) at
(i) Consider D > 0
the same time are ae 1-2,- .
(-2 m)2 - 4 1(m2 -1)>.O => 4>0 I 4
me R —(i)
(ii) Consider f (- 2) f (4) < 0 6. Conditions for Numbers fq and k2
(4 + 4m + m2 - 1) (16 - 8m + m2 - 1) < 0
(If k{ and k2 lie between the roots of/(x) = 0)
(m2 + 4m + 3) (m2 - 8m + 15) < 0

[rj
(m + l)(m + 3)(m - 3)(m - 5) < 0
(m + 3)(m + l)(m - 3)(m - 5) < 0

-X-axis X-axis

m G (-3, - 1) u(3,5) -(ii)


Hence, the values of m satisfying Eqs. (i) and (ii) at the
same time are m 6 (- 3, - 1) u (3,5).
(i) D>0

5. Conditions for Numbers k, and k2 (ii) #(^i)<0


(iii) af (k2) < 0, where a < P.
(If both roots /(x) =0 are confmed between and k2)

I •» I I I Example 40. Find the values of a for which one root


‘ a>° ! j \ 2a 4a/ of equation (a - 5)x2 - 2ax + o - 4 = 0 is smaller than 1
f(k,)' l\ \ / 7 t2 and the other greater than 2.
X-axis

777' ^.> v' "7


\ 2a 4a/
;a<0 1
I ! {I
f(k2)
Sol. The given equation can be written as

x2
2a 'I f a - 4 = 0, a
----- x + 5.
a - 5j (a -5
(i) D > 0 (roots may be equal) , ( 2a i a-4
Now, let f(x) = x2 - ----- x +
(ii) a/(fc1)>0
(iii) af<M>0 As 1 and 2 lie between the roots of /(x) = 0, we can take
(iv) fcj <----- < k2, where a < P and k{ <k2. D > 0,17(1) <0 and 1 J(2)<0.
2a
122 Textbook of Algebra

(i) Consider D > 0 i.e.,(m - l)(m - 9) > 0


/ \2 m G (- 1] U [9, oo) >••(■)
2a i -4-1. a - 4
>o /(2) > 0
a-5 7 a-5
„ i 20 i.e., 4 - 2(m - 3) + m > 0
36 a-----
V 9 >0 [va*5]
=> m < 10
(a-5)2 m G (- o°, 10) (ii)
20 and x-coordinate of vertex < 2
or a>— —(i)
9 (m-3)
i.e., <2 => m <7
(ii) Consider 1/(1) < 0 2

2a 'a-4 m G (- oo, 7) -(iii)


i2 + > 0 or a > 5 ...(ii) On combining Eqs. (i), (ii) and (iii), we get
a-5 ^a - 5, (O-5)
m G (- «>, 1]
(iii) Consider 1/(2) < 0
fa - 4 (ii) Both the roots are greater than 2
. 4a
4----------- + <0 D> 0
(a-5) ka -5?

4k
(4a-20-4a + a-4)
M<0
(^5) (0-5)

or 5<a<24 ...(iii)
-X-axis
Hence, the values of a satisfying Eqs. (i), (ii) and (iii) at the
same time are a G (5,24).

I Example 41. Let x2-(m-3)x+m = 0(meR) be a i.e. (m-1)(m - 9)>0


quadratic equation. Find the value of m for which mG(- oo, l]e[9, °o) (i)
(i) both the roots are smaller than 2. /(2) > 0
(ii) both the roots are greater than 2. i.e. 4 - 2(m - 3) + m > 0

(iii) one root is smaller than 2 and the other root is => m < 10
greater than 2. rn G (- oo, 10) (ii)
and x-coordinate of vertex > 2
(iv) exactly one root lies in the interval (1, 2).
(m-3)
(v) both the roots lie in the interval (1, 2). i.e., >2 => m > 7
2
(vi) one root is greater than 2 and the other root is
m G (7, °o) •(iii)
smaller than 1.
On combining Eqs. (i), (ii) and (iii), we get
(vii) atleast one root lie in the interval (1, 2).
m G [9,10)
(viii) atleast one root is greater than 2.
(iii) One root is smaller than 2 and the other root is
So/. Let /(x) = x2 - (m - 3)x + m
greater than 2
Here, a = l,b = - (m - 3), c = m
D > 0

w
and D~ b2 - 4ac = (m - 3)2 - 4 m
= m2 - 10m + 9 = (m - l)(m - 9)
b _ (m - 3)
and x-coordinate of vertex = - X-axis
2a" 2
(i) Both the roots are smaller than 2
D>0
i.e., (m - l)(m - 9) > 0

V\
m G (— °°, 1) u(9, °°) .(i)
/(2) < 0

i.e. 4 -2(m -3) + m < 0


m > 10
Chap 02 Theory of Equations 123

m G (10, oo) -(ii) 1 < x-coordinate of vertex < 2


On combining Eqs. (i) and (ii), we get 1<<^
i.e., <2
m G (10, <»). 2
2 < m - 3 < 4 or 5 < m < 7
(iv) Exactly one root lies in the interval (1, 2)
me (5,7) ...(iv)
D> 0

"I . I
On combining Eqs. (i), (ii), (iii) and (iv), we get
m G (j)
(vi) One root is greater than 2 and the other root is

Q
X-axis smaller than 1D > 0

i.e., (zn - l)(zn - 9) > 0


m G (- 1) u(9, °°) -(i)
■X-axis
/(l)/(2)<0

(1 - (m - 3) + m) (4 - 2(m - 3) + m) < 0
4(- zn + 10) < 0 i.e., (m - l)(zzi - 9) > 0
m - 10 >0 => m > 10 m G (- °°, 1) u(9, °o) ...(i)
•(ii) /(l)<0
zn G (10, °°)
On combining Eqs. (i) and (ii), we get i.e., 4 < 0, which is not possible.
m e (10, <») Thus, no such ‘m exists.
(v) Both the roots lie in the interval (1, 2) (vii) At least one root lie in the interval (1, 2)
D> 0 Case I Exactly one root lies in (1, 2)

h d
Hi):\ \ / \f(2)
me
me (10, “) [from (iv) part]
Case II Both roots lie in the interval (1, 2).
[from (v) part]

i.e.,
AjA2">Xads
(zzi - l)(m - 9) > 0
Hence, at least one root lie in the interval (1, 2)
m e (10, °o) u 6 or m g (10, «>)
(viii) Atleast one root is greater than 2
Case I One root is smaller than 2 and the other root
fflG (- «, 1] U [9, °°) •(i)
is greater than 2.
/(l)>0
i.e., (1 - (zn - 3) + zn) > 0 => 4 > 0 Then, mG(10, <») [from (iii) part]
mG R -(ii) Case II Both the roots are greater than 2, then
m e [9,10).
7(2) > 0
Hence, atleast one root is greater than 2.
i.e., 4 - 2(m - 3) + m > 0 => m < 10
m e (10, oo) u [9,10) or m e [9,10) u (10, «>)
m G (- oo 10) -(iii)
124 Textbook of Algebra

g Exercise for Session 3


x2 + 14x + 9 ....
1. If x is real, the maximum and minimum values of expression —=------------ will be
xz + 2x + 3
(a) 4,-5 (b)6-4 (c)-4,5 (d)-4,-5
x +2
2. If x is real, the expression takes all values in the interval
(2x2 + 3x + 6)
. , 1 1
a) —. -
\13 3
(b)[--iJ
L 13 3j
(c) -
1 1
3' 13.
(d) None of these

3. If x be real, then the minimum value of x2 -8x + 17, is


(a)-1 (b)0 (c)1 (d)2
r n
4. If the expression n?x -1 + — is non-negative for all positive real x, the minimum value of m must be .
x
<a)-; (b)0
(C)J

5. If the inequality mx + 3x -h 4 < 5 js satjsfiecj for a|| x G p then


xz + 2x + 2
(a) 1<m < 5 (b) - 1 < m < 5
(c) 1<m < 6 (d)n? < —
24
(X2 -1)
6. The largest negative integer which satisfies >0, is
(x-2)(x-3)
. (a)-4 (b) — 3
(c)-2 (d)-1
7. If the expression 2x2 + mxy + 3y2 -5y - 2 can be resolved into two rational factors, the value of |m | is
(a) 3 (b)5
(c)7 (d)9
8. If c > 0 and 4a + c < 2b, then ax2 - bx + c = 0 has a root in the interval
(a) (0,2) (b) (2,4)
(c)(0.1) (d)(- 2, 0)
9. If the roots of the equation x2 - 2ax + a22 + a - 3 = 0 are less than 3 then
(a)a<2 (b)2<a<3
(c) 3 < a < 4 (d) a > 4
10. The set of values of a for which the inequation x2 + ax + a2 + 6a < 0 is satisfied for all x e (1,2) lies in the
interval
(a) (12) (b)[1,2J
(c)[- 7,4] (d) None of these
Session 4
Equations of Higher Degree, Rational Algebraic
Inequalities, Roots of Equation with the Help of Graphs,

Equations of Higher Degree and


a a
x -1 + a2 xn~2
The equation a0 xn 4-a,i x"
or ax 3 +bx2 +cx +d = a(x -a)(x -0)(x -y)
+ ...+ an_] x +a„ =0,
where a0,a},a2,...,an_l,an are constants buta0 5*0, is a = a(x3 -Za • x2 + Za0• x -a0y)]
polynomial equation of degree n. It has n and only n roots.
(ii) For n = 4, if a, 0, y, 8 are the roots of the equation
Leta1(a2,a3,..., benroots, then
ax4 +bx3 +cx2 + dx + e = 0, where a, b, c, d, e are
• Zc*! + a2 + a 3 +...+an_1 + an=(-l)1 — constants and a * 0, then
ao
b
[sum of all roots] Za = a + 0 + y + 8=(-i)1 - = >
• Za2 a2 =a1a2 +a1a3 +...+a1an + a2 a3 + a a
+ ... +a n_1an
...+a2a„ +...+a Za0 = (a+0)(y + 8)+a0 + y8 =(-l)2 - =
a a
= (-1)2 — [sum of products taken two at a time]
Za0Y = a0(Y + 8)+Y5(a+0) =(-l)3 — = - —
a a
• Zajaaaj =(-l)3 ~ and a0y8 = (-l)4- = -
«o a a
[sum of products taken three at a time]
or ax4 + bx3 +cx2 +dx + e = a(x - a)
• «!a2a3 ...a „ = (- 1)” — [ product of all roots] (x-0)(x-y)(x-8)
«o
aD = a(x4 - Za- x3 + Za0- x 2 - Z a0y • x + a0y8)
In general, Ztti a2 a3 ...ap=(-l)p —
ao ? Example 42. Find the conditions, if roots of the
Remark equation x * - px2 + qx - r = 0 are in
1. A polynomial equation of degree n has n roots (real or
imaginary). (i) AP (ii) GP
2. If all the coefficients, i.e., a,, <% are real, then the
imaginary roots occur in pairs, i.e. number of imaginary roots (iii) HP
is always even. Sol. (i) Let roots of the given equation are
3. If the degree of a polynomial equation is odd, then atleast one A - D, A, A + D, then
of the roots will be real.
4. (x - a,)(x - ot2)(x - 03) ...(x - an) A - D + A + A + D = p =$ A =£
3
= xn + (- 1)'Za1-xn_1 + (-1)2Za,a2-x n-2
Now, A is the roots of the given equation, then it must
+... + (-1)na1a2a3...an
be satisfy
In Particular A3 - pAz + qA - r = 0
(i) For n = 3, if a, 0, y are the roots of the equation 3 / I +q(- r=0
ax3 +bx2 + ex + d = 0, where a, b, c, d are constants
P
3 I -fte P
3 3)
b b => p3 -3p3 + 9qp -27r =0
and a t 0, then Za = a + 0 + y = (~ 1)1 - = —.
a a
or 2p3 — 9pq + 27r = 0,
Za0=a0 + 0y + ya=(-l)2 - = - which is the required condition.
a a
126 Textbook of Algebra

Product of roots = (a - P)a-(a + P) = 6


(ii) Let roots of the given equation are —, A, AR, then => (2 - P)2(2 + p) = 6 => 4-p2 =3
R
P=±1
=r .'. Roots of Eqs. (i) are 1,2,3 or 3,2,1.
R
tt r , • 1111,
=> A’=r Hence, roots of the given equation are 1, - or -, -, 1.
2 3 3 2
i
A = r3 I Example 44. If cc, 0, y are the roots of the equation
Now, A is the roots of the given equation, then x3 -px2 + qx-r = 0, find
A3 - pA2 + qA - r = 0
(i) Sa2 . (ii) Sa2P- (iii) Sa3-
r-p(r)2/3=9(r),/3-r=0
Sol. Since, a, P, y are the roots of x3 - px2 + qx - r = 0.
or P(r)2,3=q(r)1/3
Sa = p,SaP = q and aPy = r
or p3r2 = q3r
(i) v Sa-Sa = p-p
or p 3r = q 3 => (a + P + Y)(a + p + Y) = P2
which is the required condition. =>a2 + P2 +Y2 +2(ap + Py + ya) = p2
(iii) Given equation is
or Sa2 +2SaP = p2
x3 - px2 + qx - r = 0 ...(i)
or Sa2 = p2 -2q
On replacing x by — in Eq. (i), then
x (ii) •/ Sa-Zap = p-q
2 /. \ => (a + P + y)-(ap + Py + Ya) - P<?
-r =0
=> a23 + a0Y + a2Y + P2a + P2Y + «Py
=> rx 3 - qx2 + px -1 = 0 (ii) + y2P + y2(x = m
Now, roots of Eq. (ii) are in AP. (a2p + a2a + p2y + p2y + y2a + y2p)
Let roots of Eq. (ii) are A - P, A, A + P, then + 3apy = pg
or Sa2P + 3r = pq
A-PA A+ A + P = — or A= —
r 3r or Sa2p = pg-3r
A is a root of Eq. (ii), then (iii) v Sa2 • Sa =(p2 -2q)-p [from result(i)]
rA3 - qA2 + pA -1 = 0
2 / v
=> (a2 + p2 + y2)(a + P + y) = p3 - 2pq
r => a3+P3+y3+(a2P + a2y+ p2a+P2y

q3 -3q3 + 9pqr -27r2 =0 + y2a + y2P) = p3 -2pg


=> 2q3 - 9pqr + 27 r2 = 0, Sa3 + Sa2p = p3 -2pq
which is the required condition. Sa3 + pq - 3r = p3 - 2pq [from result (ii)]
or Sa3 =p3 -3pq +3r
I Example 43. Solve 6x3 -11x2 + 6x-1 = 0, if roots of
the equation are in HP. I Example 45. If cc, 0, y are the roots of the cubic
Sol. Put x = — in the given equation, then equation x3+qx + r = 0, then find the equation whose
y roots are (a - P)2, (P - y)2, (y - a)2.
6 11 6 , n
—---- +_-i=o Sol. •/ a, P, y are the roots of the cubic equation
y y y
x3 + qx + r = 0 ••(■>)
=> y3 - 6y2 + lly - 6 = 0 (i)
Then, Sa = 0, SaP = q, aPy = - r •(ii)
Now, roots of Eq. (i) are in AP.
If y is a root of the required equation, then
Let the roots be a - P, a, a + p.
y = (a -p)2 =(a +p)2 -4aP
Then, sum of roots =a~P+a+a+P=6
=> 3a — 6 / o \2 4otBy
• = (a + p + y - Y)------ —
a =2 Y
Chap 02 Theory of Equations 127

= (0-Y)2 + — [from Eq. (ii)] or


Y If X is a root of the equation /(x) = 0, then /(x) is
2 4r
=> y=Y +— exactly divisible by (x - X) and conversely, if /(x) is
Y exactly divisible by (x - X), then X is a root of the
[replacing y by x which is a root of Eq. (i)]
equation /(x) = 0 and the remainder obtained is /(X).
2 4r
y = x +—
x I Example 47. If x2 + ox + 1 is a factor of ax5 +bx + c,
or x - yx + 4r = 0 ...(iii) find the conditions.
The required equation is obtained by eliminating x between Sol. v ax3 + bx + c = (x2 + ax + l)Q(x)
Eqs. (i) and (iii). Let Q(x) = Ax + B,
Now, subtracting Eq. (iii) from Eq. (i), we get then 3
ax3 + bx + c = (x2 + ax + l)(Ax + B)
(q + y) x - 3r = 0
3r On comparing coefficients of x3, x2, x and constant on
or x =------ both sides, we get
q+y
a = A, •••(*)
On substituting the value of x in Eq. (i), we get
/ . \3
0 = B + aA, -(ii)
3r 3r ' b = aB + A, ...(iii)
+ ?|
q+y ? + y, and c=B •••(iv)
Thus, y3 + 6qy2 + 9q2y + (4q3 + 27r2) = 0 From Eqs. (i) and (iv), we get
A = a and B = c
which is the required equation.
From Eqs. (ii) and (iii), a2 + c = 0 and b = ac + a are the
Remark required conditions.
Ka-P)2 = -6q, fl(a-P)2 = -(4q3 + 27r2)
I Example 48. A certain polynomial f(x),xeR, when
divided by x - a, x - b and x - c leaves remainders o, b
Some Results on Roots of a and c, respectively. Then, find the remainder when f(x)
Polynomial Equation is divided by (x-o)(x-b)(x-c), where a,b,c are
1. Remainder Theorem If a polynomial /(x) is
distinct.
divided by a linear function x - X, then the remainder Sol. By Remainder theorem f(a) = a, f(b) = b and /(c) = c
is/(X), Let the quotient be Q(x) and remainder is R(x).
i.e. Dividend = Divisor x Quotient + Remainder /(x) = (x - a)(x - b)(x - c)Q(x) + R(x)
.'. f(a) = 0 + R(a)=$ R(a) = a
Let Q (x) be the quotient and R be the remainder, thus
f(b) = 0 + R(b) => R(b) = b and /(c) = 0 + R(c)
/(x)=(x-X) Q(x) + R => R(c) = c
/(X)=(X-X)Q(X) + R = 0 + R = R So, the equation R(x) - x = 0 has three roots a, b andc. But
its degree is atmost two. So, R(x) - x must be zero
I Example 46. If the expression 2x3 + 5px2 - 4x + p polynomial (or identity).
has a remainder of 5 when divided by x + 2, find the Hence, R(x) = x.
value of p. 3. Every equation of an odd degree has atleast one real
Sol. Let /(x) = 2x3 + 3px2 - 4x + p root, whose sign is opposite to that of its last term,
/(x) = (x + 2)Q(x) + 5 provided that the coefficient of the first term is
=» /(-2) = 5 positive.
=> 2(- 2)3 + 3p (- 2)2 - 4(- 2)i + p = 5 or 13p = 13 4. Every equation of an even degree has atleast two real
p=1 roots, one positive and one negative, whose last term
is negative, provided that the coefficient of the first
2. Factor Theorem Factor theorem is a special case of
term is positive.
Remainder theorem.
5. If an equation has no odd powers of x, then all roots
Let /(x) = (x - X) Q(x) + R = (x - X) Q(x) + /(X)
of the equation are complex provided all the
If /(X) = 0, f(x) = (x - X) Q (x), therefore f(x) is coefficients of the equation have positive sign.
exactly divisible by x - X.
128 Textbook of Algebra

A (MW)
6. If x - a is root repeated m times in /(x) = 0
I \
I \
(/(*) =0 is an nth degree equation in x), then
:I + > B (PJ(P))
f(x)=(x-a)mg(x) Zi

where, g( x) is a polynomial of degree(n - m) and the root


!/w
I
/ + /+\f (p)
I LP«x.O)/p,O) \(Y.O) /SO) in
x =aisrepeated(ni - 1) time in/'(x) = 0,(m -2) times 41— X-axis
WO) \ - /Q rS (P.O)
bi f "(x) = 0,...,(m -(m -1)) times in/m-1(x) = 0.

7. Let /(x) = 0 be a polynomial equation and X, p are (v)


two real numbers.
Then, f(x) = 0 will have atleast one real root or an (AO)/
/A \(p,0) /
/A
(&0)\ (P.O)
:7(a0) \ /(Y.0) \-p' X-axis
odd number of roots between X and p, if /(X) and
/(p) are of opposite signs. wy <7 y(P)
But if /(X) and /(p) are of same signs, then either A (MW) fi(p.Hp))
(vi)
f(x) =0 has no real roots or an even number of roots
between X and p. (a) In figure (i), (ii) and (iii), /(X) and /(p) have
opposite signs and equation/(x) =0, has one,
three, five roots between X and p, respectively.
Illustration by Graphs (b) In figure (iv), (v) and (vi), /(X) and /(p) have
Since, f(x) be a polynomial in x, then graph of y = f(x) same signs and equation /(x) =0, has no, four
will be continuous in every interval. and four roots between X and p, respectively.
A(A/(A))
2 Example 49. If a,b,c are real numbers, o^O. If a is
I
root of a2x2+bx + c = 0, p is a root of
(w: + o2x2 -bx - c = 0 and 0 < a < 0, show that the
I
I equation a2x2 + 2bx + 2c = 0 has a root y that always
I (P.O)
----- a— X-axis
W.0) P\(a.O) satisfies a <y <p.
I
I
Sol. Since, a is a root of aizx2 + bx + c = 0.
p(p)
I Then, a2a2 + ba + c =0 (i)
I
I
and P is a root of a2x2 - bx - c = 0,
B (U./(H))
(0 then a2p2 - bp - c = 0 •(ii)
Let fM = a2 x2 + 2bx + 2c
I \
I \
f(a) = ai2a2 +2ba + 2c = a2a2 - 2fl2a2
'w: +\I 1 [from Eq. (i)J
I I
I 1
I 1 +\ (P.O) = - a2a2
-O----- 1----- b\(Y0) X-axis
(A.0) P\&p) rQ T —* => /(a) < 0 and /(p) = a2p2 + 2bP + 2c
(P.O) ■HP)
I
“ I = a2p2+2a2p2 [from Eq. (ii)J
B [p. '(P)] = 3a2p2
(•i)

+ 8(PJW) => /(P)>0


(x.o) / + q\ Since, /(a) and/(P) are of opposite signs, then it is clear
R/ S\ X-axis
/Wi/Ha.0) (P.O) c7(Y.0) (6.0) /(P.O) that a root y of the equation f(x) = 0 lies between a and p.
(v.O) Hence, a<y<p [vcKp]
A (MW) (iii)
B(p,f(p))
E Example 50. If a <b < c < d, then show that
f(X)<A [MW] JHp) (x-o)(x-c)+ 3(x-b)(x-d) = 0 has real and distinct
X-axis roots.
(XO) Tuo)
(M Sol. Let f(x) = (x - a)(x - c) + 3(x - b)(x - d)
Chap 02 Theory of Equations 129

Then, /(a) = 0 + 3(a - b)(a - d) > 0 [v a - b<0, a- d <0] 9. Rolle’s Theorem If /(x) is continuous function in
and f(b) = (b- a)(b ~ c) + 0<0 [v b - a > 0, b - c < 0] the interval [a, h] and differentiable in interval (a, b)
Thus, one root will lie between a and b. and /(a) = f(b), then equation /'(x) =0 will have
and f(c) = 0 + 3(c - b)(c - d) < 0 [vc - b > 0, c - d < 0] atleast one root between a and b. Since, every
and f(d) = (d - a)(d - c) + 0 > 0 d-a>0, d-c>0] polynomial /(x) is always continuous and
Thus, one root will lie between c and d. Hence, roots of differentiable in every interval. Therefore, Rolle’s
equation are real and distinct. theorem is always applicable to polynomial function
8. Let f(x) = 0 be a polynomial equation then in every interval [a, b] if f(a) = f(b).
(a) the number of positive roots of a polynomial I Example 51. If 2a+ 3b + 6c = 0 ; a,b,ceR, then show
equation /(x) =0 (arranged in decreasing order
of the degree) cannot exceed the number of
that the equation ax2 + bx + c = 0 has atleast one root
changes of signs in /(x) =0 as we move from left between 0 and 1.
to right. Sol. Given, 2a + 3b + 6c = 0
For example, Consider the equation a b
2x3 -x2 -x + l=0. -+-+c=0 (i)
3 2
The number of changes of signs from left to right Let f'(x) = ax2 + bx +c,
is 2 (+ to then - to +). Then, number of positive . ax3 bx2 .
roots cannot exceed 2. Then, f(x) = —- +----- + ex + d
3 2
(b) The number of negative roots of a polynomial .............. a b .
equation /(x) =0 cannot exceed the number of Now, /(0) = d and /(I) = - + - +■ c +' dI
'.3 2
changes of signs in /(-x). =0+d [from Eq. (i)J
For example, Consider the equation Since, /(x) is a polynomial of three degree, then /(x) is
• 5x4 + 3x3 -2x2 + 5x-8 = 0 continuous and differentiable everywhere and /(0) = /(I),
Let /(x) = 4x4+3x3-2x2+5x-8 then by Rolle’s theorem /'(*) - 0 i-e->Qx2 + bx + c = 0 has
/(-x)=5x4 -3x3-2x2 -5x-8 atleast one real root between 0 and 1.

The number of changes of signs from left to right


is (+ to -). Then number of negative roots cannot Reciprocal Equation of the Standard
exceed 1.
Form can be Reduced to an Equation
(c) If equation f(x) =0 have atmost r positive roots
and atmost t negative roots, then equation of Half Its Dimensions
/(x) = 0 will have atmost (r +1) real roots, i.e. it Let the equation be
will have atleast n -(r +1) imaginary roots, ax2m +bx2m~1 + cx 2m 2 + ...+kxm + ... + cx2+bx + a=0
where n is the degree of polynomial.
On dividing by x m, then
For example, Consider the equation
c
5x6 -8x3 + 3x5 + 5x2 +8=0 axm +bxm~1 +cxm~2 + ...+£ + ...+ —
m -2
x
The given equation can be written as b a
5x6 + 3xs -8x3 + 5x2 +8=0 +------- +---- =0
x"”1 xm
On rearranging the terms, we have
Let /(x) =5x6+3x5-8x3+5x2+8
Here, /(x) has two changes in signs. f i) ,r
a xm +----- | + b x
m -1 1
+------- +c
So, /(x) has atmost two positive real roots xm ,xm-
and /(-x) = 5x6-3x5+8x3+5x2+8 X^i -2 1
+------ + ... + k-Q
Here, /(- x) has two changes in signs. xm~2
So, J(x) has atmost two negative real roots, 1
Now, xp + 1 -I--------
1
xp H-----
n
and x = 0 cannot be root of /(x) = 0. xp + 1 xp x)
Hence, /(x) =0 has atmost four real roots,
therefore atleast two imaginary roots.
-P"1 +_L x<’-/
130 Textbook of Algebra

Hence, writing z for x + — and given to p succession the I Example 53. Solve the equation
x (12x -l)(6x - 1)(4x - 1)(3x -1) = 5.
values 1,2,3......we obtain Sol. The given equation can be written as
( 1 1 1) 5
x 2+ —= z2-2 x----- x — X------i Y x — ...ft
x2 I 12. 6 4A 3. I 12-6-4-3
x3 + —= z(z2 -2)-z=z3 -3z 11 1 x1 x1 , X 1 £
■1 1

Since, — <-< — <- and — = — —-


12 6 4 3
‘ 6 ' “ 6 12 3 4
We can introduced a new variable,
x4 + — = z(z3 -3z)-(z2 -2)=z4 -4z2 + 2
X4 1 1 1 1 f 1
y=t x---- + X — + x---- + x--
12. 6 4 l 3
and so on and generally xm + is of m dimensions in
xm 5
y = x-----
z and therefore the equation in z is of m dimensions. 24
On substituting x = y + — in Eq. (i), we get
I Example 52. Solve the equation 24
2x4 + x3 — 11x2 + x + 2 = 0. 3 1 V 1 .. 3 I 5
y + — y + — y----- y--------
24 A 24 A 24 A 24
24 J 12-6-4-3
Sol. Since, x = 0 is not a solution of the given equation. 2 2
1 3 5
On dividing by x2 in both sides of the given equation, we y2
get 24 24 12-6-4-3
2 ( x2 + -M + £ Hence, we find that
-11 = 0 •••ft)
I x2) x .2
y 24! -
Put x + — = y in Eq. (i), then Eq. (i) reduce in the form
x 7 , 7
2(y2 -2) + y-ll = 0 i.e. y. = — and y? =-----
24 24
2y2 + y- 15 = 0 Hence, the corresponding roots of the original equation are
o j 5 1,1
yi = -3andy2 = - ----- and —.
Ct 12 2
Consequently, the original equation is equivalent to the Type II An equation of the form
collection of equations
(x - d)(x - b)(x - c)(x - d) = Ax2
x+-=-3
.x where, ab = cd. can be reduced to a collection of two
£_ quadratic equations by a change of variable y = x + —.
X x
-3-75 1
we find that, Xj = —^•X2= . x3 = -, x4 = 2 I Example 54. Solve the equation
2
(x + 2)(x + 3)(x + 8)(x + 12) = 4x2.
Equations which can be Reduced Sol. Since, (- 2)(- 12) = (- 3)(- 8), so we can write given equa­
tion as
to Linear, Quadratic and Biquadratic (x + 2)(x + 12)(x + 3)(x + 8) = 4x2
Equations => (x2+14x + 24)(x2+llx + 24) = 4x2
Type I An equation of the form
Now, x = 0 is not a root of given equation.
(x - a)(x - b)(x - c)(x - d) = A
On dividing by x2 in both sides of Eq. (i), we get
where, a <b <c <d, b ~a-d ~c, can be solved by a ( 24 , ' 24 A
change of variable. x + —+ 14 x + —+ 11 =4 (ii)
I x x J
_ (x - a) + (x - 6) + (x - c) + (x - d) 24
i.e. y~ 4 Put x + — = y, then Eq. (ii) can be reduced in the form
x
(a + b + c + d) (y + 14)(y + 11) = 4 or y2+25y + 150 = 0
y = x - -------------------
4 yj = -15 and y2 = -10
=a

Chap 02 Theory of Equations 131

Thus, the original equation is equivalent to the collection of P(x)>0, Q(x)>0


equations ^|l>0=>{P(x)Q(x)>0=>
(1) or
x + —= -15,
X P(x) <0, Q(x) <0

X + —= -10, P(x)>0, Q(x)<0


X
(2) ^<0=>{P(x)Q(x)<0=> or
x2 + 15x4-24 =0
i.e. P(x) <0, Q(x) >0
x2 + lOx + 24 = 0
P(x)>0, Q(x)>0
On solving these collection, we get P(x)Q(x)>0
-15- "129 -15 + V129 (3) ^>o=J or
---- ,. x.2 =
*i =------------ x3 = - 6, x4 = - 4 £>(x) 1. Q(x)*0 P(x)<0, Q(x)<0
2 2
Type III An equation of the form (x -a)4 +(x-b)4 =A
P(x) >0, Q(x) <0
P(x)Q(x)<0
can also be solved by a change of variable, i.e. making a (4) or
substitution y = ——+^X——. Q(x)*0
P(x)<0, Q(x)>0
2

i Ii Example 55. Solve the equation I Example 56. Find all values of a for which the set of
(6-xr + (8-x)4 =16. all solutions of the system
Sol. After a change of variable,
x2 + ox-2
—------------ <2
(6-x) + (8-x) x -x + 1
y 2
x2 + ox-2
y = 7 - x or x =7 - y _ >—5
X -x + 1
Now, put x = 7 - y in given equation, we get
(y-i)4+(y + l)4=16
is the entire number line.
Sol. The system is equivalent to
y4 + 6y2 - 7 = 0
x2 - (a + 2)x + 4
(y2 + 7)(y2 - 1) = 0 >0
x2 - x + 1
y2 + 7 *0 4x2 +(a-3)x + 1 > o
[y gives imaginary values] x2 - x + 1
y22 - 1 = 0 1 2 3
Since, x2 - x + 1 = x — | + — > 0, this system is
Then, yi = - 1 and y2 = 1 2
Thus, x, = 8 and x2 = 6 are the roots of the given equation. x2 - (a + 2)x + 4 > 0
equivalent to
4x2 +(a - 3)x + 1 > 0

Rational Algebraic Inequalities Hence, the discriminants of the both equations of this
system are negative.
Consider the following types of rational algebraic (a + 2)2 - 16 < 0
i.e., => (a + 6)(a - 2) < 0
inequalities (a-3)2 - 16 <0

P(x) A P(X) n +__\ +


—— <0,
Q(X) QM -6 2

i.e., xe(-6,2) ...(i)


>0, => (a + l)(a-7)<0
e(x) 1 «x)
If P(x) and Q(x) can be resolved in linear factors, then use
Wavy curve method, otherwise we use the following
i.e. xg(-1, 7) (ii)
statements for solving inequalities of this kind.
Hence, from Eqs. (i) and (ii), we get
x e (-1,2)
132 Textbook of Algebra

The first system of collection Eq. (i) is equivalent to the


Equations Containing system of collection
Absolute Values 2x - 4 - 2x = 4, if 2x > 4
By definition, | x | = x, if x > 01 x | = - x, if x < 0 - 2x + 4 - 2x = 4, if 2x < 4
- 4 = 4, if x > 2
I Example 57. Solve the equation x2 - 5|x| + 6 = 0.
- 4x = 0, if x < 2
Sol. The given equation is equivalent to the collection of
The first system is failed and second system gives x = 0.
systems
(x - 2)(x - 3) = 0, if x > 0 Hence, x = 0 is unique solution of the given equation.
x2 - 5x + 6 = 0, if x > 0
x2 + 5x + 6 = 0, if x < 0 (x + 2)(x + 3) = 0, if x < 0
Hence, the solutions of the given equation are
Important Forms Containing
x ] = 2, x 2 = 3, x j — — 2, x 4 = — 3 Absolute Values
Form 1 The equation of the form
I Example 58. Solve the equation
|/(x) + g(x)| = |f(x)| + |g(x)|
x2 -8x + 12 x2 -8x + 12
is equivalent of the system
x2 -10x + 21 x2 -10x + 21
f(x)g(x)>Q.
Sol. This equation has the form |/(x)| = - /(x)
x2 -8x +12
I Example 60. Solve the equation
when, f(x) = x X2
x2 - lOx + 21 + |X| =
such an equation is equivalent to the collection of systems
x-1 ix-ir
f(x) = -/(x),if/(x)>0 Sol. Letf(x)-
X
and g(x) = x,
f(x) = /(x), if/(x)<0 x-1

The first system is equivalent to /(x) = 0 and the second X +x- x2


Then, f(x) + g(x) =
system is equivalent to f(x) < 0 the combining both x-1 x-1
systems, we get
.•. The given equation can be reduced in the form
x22 -8x + 12 l/WI + |gWI = l/W + gWI
—-------------- <0 Hence, f(xH(x)>0
x2 - lOx + 21
(x-2)(x-6) J^>0
(x-3)(x-7) x-1

+
+ + +
'0 1
2 3 6 7

From Wavy curve method, x G (1, <») U {0}.


Hence, by Wavy curve method,
xG [2,3) U[6,7)
Form 2 The equation of the form
I Example 59. Solve the equation 1/1 (x)l + 1/2 (x)l + ...+!/„ (x)l = gW •••(»
|x-|4 - x||-2x = 4.
where, f\ (x), f2(x),..., fn (x), g(x) are functions of x and
Sol. This equation is equivalent to the collection of systems
g(x) may be constant.
|x - (4 - x)| - 2x = 4, if 4 - x > 0
Equations of this form solved by the method of
|x + (4 - x)| - 2x = 4, if 4 - x < 0
intervals. We first find all critical points of
|2x-4|-2x = 4, if x < 4 fi(*)> ..... fn(*)> coefficient of x is positive, then
=>
4 ~2x = 4, if x > 4 graph start with positive sign (+) and if coefficient of x is
The second system of this collection negative, then graph start with negative sign (-). Then,
gives x=0 using the definition of the absolute value, we pass from
but x>4 Eq. (i) to a collection of systems which do not contain the
Hence, second system has no solution. absolute value symbols.
Chap 02 Theory of Equations 133

I Example 61. Solve the equation For —-<0, ifx>0


x+1
|x-1| + |7-x| + 2|x-2| = 4.
Sol. Here, critical points are 1, 2, 7 using the method of inter­
vals, we find intervals when the expressions x - 1,7 - x
and x - 2 are of constant signs.
0<x<l ...(i)
i.e. x<l, l<x<2,2<x<7, x>7
For — <0, ifx<0
+ + + x-1
(x-1) 1
-- (7-x) + + +>
7
I - 1 < x <0 -(ii)
(x-2) ' 4- Hence, from Eqs. (i) and (ii), the solution of the given
2
equation is x e [- 1,1}
Thus, the given equation is equivalent to the collection of Alite r
four systems, 1
1-_LL >- => >1
x<1 x<1 1 + 1*1 2 l + |x| 2
[-(x - 1) + (7 - x) — 2(x - 2) = 4 x=2 1
[l<x<2 >- => or |x| £ 1
l<x<2 2
|(x - 1) + (7 - x) - 2(x - 2) = 4
1 + 1*1
x=3 - 1 < x < 1 or x => [-1,1]
|2<x<7 2<x<7
[(x - 1) + (7 - x) + 2(x - 2) = 4
Jx>7
x=l
x>7
Equations Involving Greatest Integer,
[(x - 1) - (7 - x) + 2(x - 2) = 4 x=4
Least Integer and Fractional Part
From the collection of four systems, the given equation has 1. Greatest Integer
no solution.
[x] denotes the greatest integer less than or equal to x i.e.,
[x] < x. It is also known as floor of x.
Inequations Containing Thus, [3.5779] = 3, [0.89] = 0,[3] = 3
Absolute Values [- 8.7285] = -9
By definition, |x|<a=>-a<x<a(a>0)
[-0.6] = - 1
|x|<a =>-a<x <a
[-7] = -7
|x|>a=>x<-a and x > a
In general, if n is an integer and x is any real number
and |x|>a=>x<-a and x > a.
between n and n +1
|X|
I Example 62. Solve the inequation 1 -
1 + |x|
>12 i.e. n<x <n + l, then[x] = n

Sol. The given inequation is equivalent to the collection of Properties of Greatest Integer
systems (i) [x ±n] = [x]±n,ne I
i—L_ > -, if x>0 ,1 (ii) [-x] = -[x],xe/
>-,if x>0
i+X 2 , . I1 + xl 2
(iii) [—x] = — 1 — [x], x £ I
i+— > -, if x < 0 > —, if x < 0 (iv) [x]-[-x] = 2n» if x = n,ne I
l-x 2 |l-x| 2
(v) [x] -[- x] = 2n +1, if x = n + {x}, n G I and 0 < {x} < 1
1 1-x
>-, if x>0 > 0, if x > 0
1+x 2 1+x
=> =>
1 (vii) [x] > n => x >n +1, n G I
> -, if x < 0 — > 0, if x < 0
1-x 2 1-x (viii)[x] <n=>x<n + l,nGl
L < 0, if x > 0 (ix) [x] < n => x < n, n e I
x+1
=> (x) n2 <[x]<n, =>n2 <x<n} + l,nI,n2el
x+1
— <0. if x<0
x -1 (xi) [x + y]>[x]+[y]
134 Textbook of Algebra

2. Least Integer
(x) or x"| denotes the least integer greater than or equal
n+l n +2 n+4 n +8
(xiii) + + + +... = n, n G N to x i.e., (x) > x or |~x"| > x. It is also known as ceilling
2 4 8 16
of x.
1 2 n-1
(xivXx] + + + ... + = [nx], Thus, (3.578) = 4 , (0.87) = 1,
n n n
ne N (4) = 4
Graph of y = [x] 8.239~|=— 8, [*—0.7~|= 0
y
3 In general, if n is an integer and x is any real number
I
between n and n + l
2
i.e., n <x <n + 1, then(x) =n + 1
1
till
iiii
X [x] = n x=[x] = n+1
I I
? 2 3 4

-2 o-
n x
M- H
-3
Relation between Greatest Integer and Least Integer
Remark [x], XGl
(X) =
Domain and Range of [x] are Rand /. respectively. [x] + l, X<£l

I Example 63. If [x] denotes the integral part of x for i.e. If x G/, then x =[x] = (x). [remember]
real x, then find the value of Remark
[>r+ '1........... _L + 1 JL■’ If (x) = n, then (n-1) < x < n
[4j |_4 200 4 100 .4 200
’1 199” Graph of y = (x) = pc"]
+... + —I------
.4 200. y
Sol. The given expression can be written as sin (-1)
1 1 2 1 3 3
+ - + — + - +---- I
I
i
i
200 4 200 4 200 2 — I i
i
I i
1 199 I i
+ ...+ - +---- 1
I
I
i
i
4 200 I i
-2 -1 I i

1 X' -i—<?— ■X
= 200— =[50] =50 [from property (xiv)] I
I
• I
I
2 3
4

+y
I Example 64. Let [a] denotes the larger integer not
exceeding the real number a If x and y satisfy the
equations y = 2[x] + 3 and y = 3[x - 2] simulaneously, Remark
determine [x + y]. Domain and Range of (x) are R and [x] + 1, respectively.

Sol. We have, y = 2[x] + 3 = 3[x-2] ...(i)


2[x] + 3 = 3( [x] - 2) [from property (i)]
I Example 65. If [x] and (x) are the integral part of x
2[x] + 3 = 3[x] — 6 and nearest integer to x, then solve (x)[x] = 1.
[x] = 9 Sol. Case I If x G I, then x = [x] = (x)
From Eq. (i),y = 2x9 + 3 = 21 Given equation convert in x2 =1.
[x + y] = [x + 21] = [x] + 21 = 9 + 21 = 30. x = (± 1)
Hence, the value of [x + y] is 30. Case II If x G I, then(x) = [x] + 1
Chap 02 Theory of Equations 135

/.Given equation convert in I Example 67. If {x} and [x] represent fractional and
([x] + l)[x] = l => [x]2+[x]-l=0 integral part of x respectively, find the value of
or [x] =
-i±7s [impossible] . . 2S° {x + r}
2 x 2000'
Then, final answer is x = ± 1.

I Example 66. Find the solution set of So/. [x]+2f 2000


=w+ . 2000
[from property (i)]
(x)2 + (x +1)2 = 25, where (x) is the least integer r — I r = i

greater than or equal to x. = [x] + — 71 = [x] + X 2000 = [x] + {x} = x


Sol. Case I If x e I, then x = (x) = [x] 2000 r — i 2000

Then, (x)2 + (x + l)2 = 25 reduces to


....... 2 I Example 68. If {x} and [x] represent fractional and
x2 + x + 1 = 25 => 2x2 + 2x - 24 = 0 integral part of x respectively, then solve the equation
=> x2 + x-12 = 0 => (x +4)(x-3) = 0
x-1 = (x-[x])(x-{x}).
x = -4,3 ...(i)
Case II If x g I, then (x) = [x] + 1 Sol. X = [x] + {x}, 0 < {x} < 1
Then, (x)2 + (x + I)2 = 25 reduces to Thus, given equation reduces to
{[x] + I}2 + {[x + 1] + I}2 = 25 [x] + {x}-l = {x}[x]
=> (M+1)! + {[x] + 2}2=25 => {x}[x]-[x]-{x} + l=0
=> 2[x]2 + 6[x]-20 = 0 ([x]-l)({x}-l) = 0
=> [x]2 + 3[x]- 10 = 0 Now, {x} - 1 * 0 [■■•0< {*}<!]
{[x] + 5H[x]-2} = 0 [x]-l = 0
[x] = - 5 and [x] = 2 [x]= 1
=> xe[-5,-4)u[2,3) xe[l,2)
x 11,
xg(-5,-4)u(2,3)
Problem Solving Cycle
(ii)
If a problem has x, | x |,[x], (x), {x}, then first solve |x|,
On combining Eqs. (i) and (ii), we get
x 6 (-5, - 4] u(2,3] then problem convert in x,[x],(x), {x}.

3. Fractional Part X. I X I. [x]. (X). {X

{x} denotes the fractional part of x, i.e.O < {x} < L X = [X] + {X} X, [x], (X), {x}
Thus, {2 • 7} = 0.7, {5} = 0, {- 3.72} = 0.28
If x is a real number, then x = [x] + {x}
i.e., x = n + f, where n G I and 0 < f < 1
Properties of Fractional Part of x
[X]. {X}
(i) {x ±n} = {x}, ne I (ii) IfO < x <1, then {x} = x X. [X], {X}
Graph of y = {x}

[x], XGl
Secondly, solve (x) =
[x] +1, X € I
X' ■X Then, problem convert in x,[x], {x}.
-3 -2 01T 1 2 3 4
Now, put x=[x] + {x}
Remark Then, problem convert in[x] and {x}.
1. For proper fraction 0 < {x } < 1. Since, 0 < {x} < 1, then we get [x]
2. Domain and range of {x} are R and [0,1), respectively.
From Eq. (i), we get {x}
3. {-5.238} = {-5-0238} = {-5-1 + 1 -0.238}
= {- 6+ 0.762}= {6.762} = 0.762 Hence, final solution is x = [ x ] + {x}.
136 Textbook of Algebra

I Example 69. Let {x} and [x] denotes the fractional I Example 71. Solve the equation ?
and integral parts of a real number x, respectively. (x)2=[x]2 + 2x /
Solve4{x} = x + [x]. where, [x] and (x) are integers just less than or equal
Sol. v x = [x] + {x} •■•(0 to x and just greater than or equal to x, respectively.
Then, given equation reduces to Sol. Case I If x G I then
4{x} = [x]+{x} + [x] x = [x] = (x)
2
W=^[x] -(ii) The given equation reduces to
x2 = x2 + 2x
2 r , 3
0 < {x} < 1 =>0 < —[x] < 1 or 0 < x < - => 2x = 0 or x = 0 ...0)
2
Case II If x £ I, then (x) = [x] + 1
[x] = 0,1
2 The given equation reduces to
From Eq. (ii), {x} = 0, -
([x] + I)2 = [x]2 + 2x
2 5 1 = 2(x — [x]) or {x} = |
From Eq. (i), x = 0,1 + - i.e., x = 0, -
r -i 1 1
x = [x] + -=n + -,nGl .(ii)
I Example 70. Let {x} and [x] denotes the fractional 2 2
1
Hence, the solution of the original equation is x = 0, n +
and integral part of a real number (x), respectively. n G I. 2
Solve |2x-1| = 3[x] + 2{x}.
Sol. Case I 2x - 1 > 0 or x > -
I Example 72. Solve the system of equations in x,y
2 and z satisfying the following equations:
Then, given equation convert to
x+[y] + {z} = 3-1
2x - 1 = 3 [x] + 2{x} ...(i)
x = [x] + {x} •••(ii) {x}+ y + [z] = 4 • 3
From Eqs. (i) and (ii), we get [x] + {y}+z = 5-4
2([x] + {x}) - 1 = 3[x] + 2{x} where, [■] and {•} denotes the greatest integer and frac­
[x] = -l tional parts, respectively.
-l<x<0 Sol. v [x] + {x} = X, [y] + {y} = y and [z] + {z} = z,
No solution V X>-
2 On adding all the three equations, we get
Case II 2x-l<0orx<- 2(x + y + z) = 12.8
2
Then, given equation reduces to => x + y + z =6.4 ■CO
1 - 2x = 3[x] + 2{x} Now, adding first two equations, we get
-(iii)
x = [x] + {x} -(iv) x + y+ z + [y] + {x}=7.4
From Eqs. (iii) and (iv), we get => 6.4 + [y] + {x} = 7.4 [from Eq. (i)]
l-2([x] + {x}) = 3[x] + 2{x} [y] + {x}=i
l-5[x] = 4{x} [y] = 1 and {x} = 0 .(ii)
On adding last two equations, we get
x + y + z + {y} + [z] = 9.7
4
Now, {y} + [z] =3.3 [from Eq. (ii.)]
l-5[x] :. [z] = 3 and {y} = 0.3 -.(iii)
0<
4 On adding first and last equations, we get
0<l-5[x]<4 x + y + z + [x] + {z} = 8.5
0>-l + 5[x]> -4 [x] + {z} = 2.1 [from Eq. (i)|
3 r
1 > 5[x] > - 3 or [x] = 2, {z} = 0.1 ...(iv)

[x] = 0 From Eqs. (i), (ii) and (iii), we get


x = [x] + {x} = 2 + 0 = 2
From Eq. (v), {x} = —
4 y = [y] + {y) = i+0.3= 1.3
n 1 1 and z = [z] + {z} = 3 + 0.1= 3.1
x=0+-=-
4 4
Chap 02 Theory of Equations 137

Roots of Equation with 1 < x<2


We have, f(x) = x 3- 3 and g(x) = 1
the Help of Graphs or x3-3=l => x3= 4
Here, we will discuss some examples to find the roots of x = (4),/3
equations with the help of graphs. Hence, x = 41 '3 is the solution of the equation x3 - [x] = 3.

Important Graphs Aliter


x = [x] + f,Q< f <1,
1. y = ax3 + bx2 + ex + d
Then, given equation reduces to
x3~(x-f) = 3 => x’-x = 3-/
Hence, it follows that
2<x33-x<3
=> 2 < x(x + 1) (x - 1) < 3
Further for x > 2, we have x(x + 1) (x - 1) > 6 > 3
2. x = ay3 + by2 + cy + d For x < - 1, we have x(x + l)(x - 1) <0 < 2
For x = -1, we have x(x + l)(x - 1) = 0 < 2
For-1 < x < 0, we have x(x + l)(x - 1) < - x < 1
and forO < x < 1,we have x(x + l)(x - 1) < x < x3 < 1
Therefore, x must be 1 < x < 2
[x] = l
Now, the original equation can be written as

n
3. y = ax4 + bx 3 + c x2 + dx + e x3-1 = 3 => x3=4
Hence, x = 41 z 3 is the solution of the given equation.

I Example 74. Solve the equation x5 - 3x - a = 0 for


different values ofo.
Sol. We have, x3-3x-a = 0 => x3-3x = a
a >0 a <0
Let/(x)= x3-3x and g(x) = a

I Example 73. Solve the equation x33-[x] = 3, where f'(x) = Q


[x] denotes the greatest integer less than or equal tox. 3x2 - 3 = 0

Sol. We have, x3 - [x] = 3 => x = -1,1


f"(x)='6x
=> x3-3 = [x]
Y .
Let /(x)= x3-3andg(x) = [x} —2 /
It is clear from the graphs, the point of intersection of
two curves y =■ f(x) and y = g(x) lies between (1,0) and y=a
(2,0).
Y y = f(x)
■X
2 -J3) 0
y = gW
1 ---- 3

X' X
1 2 3
y = x3 - 3x
Y
-2,
... /"(-i) = -6<0 and /z'(l) = 6>0
-3
.’./(x) local maximum at x = (-1) and local minimum at
x = land /(-I) = 2 and /(I) = -2 andy = g(x) = a is a
Y' straight line parallel to X-axis.
138 Textbook of Algebra

Following cases arise I Example 76. Find all values of the parameter k for
Case I When a > 2, which all the roots of the equation
In this case y = /(x) and y = g(x) intersects at only one x° + 4x3 -8x2 + k = 0are real.
point, so x3 - 3x - a = 0 has only one real root.
Sol. We have, x4+ 4x3-8x2+ k = 0
Case II When a = 2,

a
i fy
In this case y = /(x) and y = g(x) intersects at two points,
sox3-3x-a = 0has three real roots, two are equal and x- -X
one different. I
T

Case III When -2 < a < 2,


I
I
y = g(x)
I

In this case y = /(x) and y = g(x) intersects at three points,


I
I
-a
I
sox3-3x-a = 0 has three distinct real roots. I
I
I
I
Case IV When a = - 2, I
I
In this case y = f(x) and y = g(x) touch at one point and I
I 'y = fix)
I
intersect at other point, so x3- 3x - a = 0 has three real I
I
I
roots, two are equal and one different. I
128
Case V When a < - 2,
In this case y = f(x) and y = g(x) intersects at only one ir
point, sox3-3x-a = 0 has only one real root. => x4+4x3-8x2=-fc
Let _f(x) = x4 + 4x3-8x2 and g(x) = -t
I Example 75. Show that the equation
P(x) = 0
x3+ 2x2+ x+ 5 = 0 has only one real root, such that 3
4x3 + 12x2 - 16x = 0 =>x = -4,0,l
[a] = - 3, where [x] denotes the integral point of x .
and /zz(x) = 12x2+24x - 16
Sol. We have, x3 + 2x2 + x + 5 = 0
/zz(-4) = 80, fzz(0) = -16,/zz(l) = 20
x3+2x2 + x = -5 .-./(x) has local minimum at x = - 4 and x = 1 and local
Let /(x) = x3 +2x2 + x and g(x) = -5 maximum at x = 0
f'(x) = 0 => 3x2 + 4x + 1 = 0 and /(-4) = -128, /(0) = 0, /(l) = -3.

=> . x = -l,-i and /zz(x) = 6x + 4 Following cases arise


Case I When - k > 0 i.e., k < 0
.*. /zz(-l) = —2<0 and f -2+ 4 = 2>0 In this case y = x4 + 4x3 - 8x2and y = (-k) intersect at
two points, so x4 + 4x3-8x2+fc = 0has two real roots.
.’./(x) local maximum at x = -1 and local minimum at
1 Case II When -k = 0 and -k = - 3, i.e. k = 0,3
x=— In this case y = x4 + 4x 3-8x2 and y = - k intersect at four
and 3 r n
1
.f(-l) = 0,f -- =
4 points, sox4+4x3-8x2 + fc=0 has two distinct real roots
3 27 and two equal roots.
Y Case III When -3 < - k < 0, i.e. 0 < k < 3
In this case y = x4 + 4x3-8x2 and y = -k intersect at four
1_ y = /(x)=x3 + 2x2+x distinct points, so x4 + 4x3 - 8x2 + k = 0 has four distinct
"3
X' ----------------------- ►X real roots.
0
Case IV When -128 <-k < -3, i.e. 3 < k <128
In this casey = x4 + 4x3 - 8x 2andy = -k intersect at two
distinct points, sox4 + 4x3~8x2+fc=0 has two distinct
real roots.
Case V When -k = -128 i.e., k = 128
y =g(x)=-5 In this casey = x4 + 4x3 - 8x 2andy = -k touch at one
point, sox4 + 4x3-8x2+fc = 0 has two real and equal roots.
and /(-2) = -2 and /(-3) = -12 Case VI When -k < -128, i.e. Ac > 128
Therefore, x must lie between (-3) and (-2). In this casey = x4+4x3-8x2 andy = -k do not
i.e. -3 <a <-2 => [a] =-3 intersect, so there is no real root.
Chap 02 Theory of Equations 139

I Example 77. Let -1 < p < 1, show that the equation We observe that, the line y = g(x) = p, where -1 < p < 1
intersect the curve y = f(x) exactly at point ct G -, 1 .
4x3 - 3x -p = 0 has a unique root in the interval 1 |_2
Hence, 4x3- 3x - p = 0has exactly one root in the interval
and identify it.
Sol. We have, 4x3 - 3x - p = 0 [ill
2
=> 4x3-3x = p Now, we have to find the value of root a.
Let /(x) = 4x3-3x andg(x) = p Let a = cos0, then 4 cos3 0 - 3cos0 - p = 0
1
f'(x) = 0 => cos30 = p => 30 = cos-1(p) or 0 = -cos”’(p)
12x2 - 3 = 0
fl i 1 33
a = cos0 = cosl-cos (pYf
1 3
x= -- and /"(*)= 24x
2 2 Aliter
Is! . Let 0(x) = 4x3 - 3x - p
j = -12 < Oand f = 12 > 0
2J 1
0'(x) = 12x2 - 3= 12 x + - x—
1
1
.’./(x) has local maximum at x = - - and local minimum
2
at | x = - I
Al 1"! 44 3,.
3 /l^
1 4 1
Also, f -- = — + - = 1 and f - = ’=-1 Clearly, 0'(x) > 0 for x G 1 .
I 2 J 88 22 <2j
2 8 2 2

y Hence, 0(x) can have atmost one root in -, 1


2
1 y = 4*) Also, 0| - j = -1 - p and 0(1) = 1 - p
k 2J
y = gW ••• 4; «D = -(1-P2)=(P!-DSO [V-1<P<1]
\ z

■x Since, 0(x) being a polynomial, continuous on r‘1-l and,


J3 2'
0^- j 0( 1) < 0. Therefore, by intermediate value theorem
0l - 10(1) < 0. Therefore, by intermediate value theorem

0(x) has atleast one root in -, 1 .

Hence, 0(x) has exactly one root in 1 .


y'
140 Textbook of Algebra

g Exercise for Session 4


1+ a
1. If a, p, y are the roots of x 3 - x 2 -1 = 0, the value of £ , is equal to
1-a !
(a)-7 (b)-6
(c)-5 (d)-4
2. If r, s, t are the roots of the equation 8x 3+ 1001x + 2008 = 0. The value of
(r + s)3+ (s + f)3+ (t + r)3is
(a)751 (b) 752
(c) 753 (d) 754
3. If a,p,Y. 8 are the roots of equation x4 + 4x 3-6x2 + 7x - 9 = O,the value of 11(1+ a2) is
(a) 9 (b)11
(013 (d) 15
4. \fa,b,c,d are four consecutive terms of an increasing AP, the roots of the equation
(x -a)(x — c) + 2(x -b)(x ~d) = 0 are
(a) non-real complex (b) real and equal
(c) integers (d) real and distinct
5. If x2 + px + 1 is a factor of the expression ax 3+ bx + c then
(a)a2-c2=ab (b)a2+c2=-ab
(c)a2-c2=-ab (d) None of these

6. The number of real roots of the equation x2 - 3| x | + 2 = 0 is


(a) 1 (b)2
(c)3 (d)4
7. Let a / 0 and p (x) be a polynomial of degree greater than 2, if p (x) leaves remainder a and (-a) when divided
respectively by x + a and x - a, the remainder when p (x) is divided by x2 - a2, is
(a) 2x (b) -2x
(0 * (d) —x
8. The product of all the solutions of the equation (x - 2)2 - 31 x - 21 + 2 = 0 is
(a) 2 (b) —4
(c) 0 (d) None of these
9. IfO < x < 1000 and = |^x,where [x] is the greatest integer less than or equal to x.the

number of possible values of x is


(a) 32 (b) 33
(c) 34 (d) None of these
10. If [x] is the greatest integer less than or equal tox and (x)be the least integer greater than or equal to x and
[x]2 + (x)2 > 25 then x belongs to
(a) [3 4] (b)
(c)[4,«) (d) (-«,-4]u [4, oo)
Session 5
Irrational Equations, Irrational Inequations, Exponential
Equations, Exponential Inequations, Logarithmic Equations,
Logarithmic Inequations

Irrational Equations This equation is defined for 2x + 7 > 0


Here, we consider equations of the type which contain the x>-’
and x + 4 > 0 => 2
unknown under the radical sign and the value under the
x>-4
radical sign is known as radicand. x>-’
• If roots are all even (i.e. Vx, Vx, Vx,.., etc) of an equation 2
7
are arithmetic. In other words, if the radicand is negative For x > the left hand side of the original equation
(i.e.x<0), then the root is imaginary, if the radicand is
is positive, but right hand side is zero. Therefore, the
zero, then the root is also zero and if the radicand is equation has no roots.
positive, then the value of the root is also positive.
(ii) We have, ^(x - 4) = -5
• If roots are all odd (i.e. Vx, Vx, Vx,... etc) of an equation,
The equation is defined for x - 4 > 0
then it is defined for all real values of the radicand. If the
radicand is negative, then the root is negative, if the x>4
radicand is zero, then the root is zero and if the radicand For x > 4, the left hand side of the original equation is
is positive, then the root is positive. positive, but right hand side is negative.
Therefore, the equation has no roots.
(iii) We have, ^/(6 - x) - -Jx - 8 = 2
Some Standard Formulae to The equation is defined for
Solve Irrational Equations 6 - x > 0 and x - 8 > 0
If f and g be functions of x, k E N. Then, x<6
x>8
1- 2V72Vg =2li/fi,f>0,g>0
Consequently, there is no x for which both expressions
2.2k4f /a4g=2k/f^.f^g>0 would have sense. Therefore, the equation has no roots.
(iv) We have, - x) = ^/(x - 7)
3-l/l2Vi=W:g).«20 This equation is defined for

2vw=2Vm / 2vnnjg a o, s * o -2-x>0 => x<-2


For x < - 2 the left hand side is positive, but right
hand side is negative.
^g=2k/f\2k4gJs^
Therefore, the equation has no roots.
S Example 78. Prove that the following equations has (v) We have, -Jx + ^(x + 16) = 3
no solutions. The equation is defined for
(i) ^(2x + 7) + ^(x + 4) = 0 (ii) ^/(x - 4) = -5 x>0
x > 0 and x + 16 > 0 =>
c2^x=>/(x^7) x>-16

(vi) 7jx+B-Fx+^=9S Hence, x>0


x5 For x > 0 the left hand side > 4, but right hand side is
3. Therefore, the equation has no roots.
(vii) J(x-3) -^/x+9 = 7(x-1)
(vi) We have, 7>/x + 8v-x + — = 98
Sol. (i) We have, ^(2x + 7) + -J(x + 4) = 0 x
142 Textbook of Algebra

For x < 0, the expression 7>/x is meaningless, Sol. We have, 3^(x + 3) - -Jx - 2 = 7
For x > 0, the expression sj-x is meaningless => 37(x + 3) =7 + 7(^-2)
and for x = 0, the expression is meaningless. On squaring both sides of the equation, we obtain
x 9x 4- 27 = 49 4- x - 2 + 14^/x - 2
Consequently, the left hand side of the original => 8x - 20 = 14^/(x — 2)
equation is meaningless for any x e R. Therefore, the
equation has no roots. (4x - 10) = 7y]x-2
Again, squaring both sides, we obtain
(vii) We have, ^(x - 3) - y](x 4- 9) = ^/x - 1
16x2 4- 100 - 80x = 49x - 98
This equation is defined for
16x2 - 129x 4- 198 = 0
x-3>0 x >3
( 33^
• x + 9 >0 => x>-9 (x-6) x-— =0
I 16)
x-1>0 x>1 33
X[ = 6 and x2 = —
x.
Hence, x>3 16
33
For x >3,y/x-3 < Jx+ 9 i.e. J(x-3) - 7(* + 9) < 0 Hence, X] = 6 satisfies the original equation, but x2 = —
16
Hence, for x > 3, the left hand side of the original does not satisfy the original equation.
equation is negative and right hand side is positive. 33
/. x2 = — is the extraneous root.
Therefore, the equation has no roots. 16
Form 2 An equation in the form
Some Standard Forms to 2n4fM=gW,neN
Solve Irrational Equations is equivalent to the system
#(x)>0
Form 1 An equation of the form f(x)=g2n(x)
f2n(x) = g2n(x), ne N is equivalent to J(x) =g(x).
I Example 81. Solve the equation
Then, fmd the roots of this equation. If root of this
equation satisfies the original equation, then its root of the
7(6-4x-x2) = x + 4.
original equation, otherwise, we say that this root is its Sol. We have, J(6 - 4x - x2) = x + 4
extraneous root.
This equation is equivalent to the system
x + 4>0
Remark
Squaring an Equation May Give Extraneous Roots > 6 - 4x - x2 = (x + 4)2
Squaring should be avoided as for as possible. If squaring is x>-4
necessary, then the roots found after squaring must be checked
x 4- 6x + 5 = 0
whether they satisfy the original equation or not. If some values
of x which do not satisfy the original equation. These values of x On solving the equation x2 4- 6x 4- 5 = 0
are called extraneous roots and are rejected.
We find that, xt = (-1) and x2 = (-5) only xt = (-1) satisfies
8 Example 79. Solve the equation V7 = x - 2. the condition x > - 4.
Consequently, the number -1 is the only solution of the
Sol. We have, Vx = x - 2 given equation.
On squaring both sides, we obtain
Form 3 An equation in the form
x = (x - 2)2
(i)
x2-5x4-4=0 => (x-l)(x-4) = 0
where f(x), g(x) are the functions of x, but h(x) is a
Xj = 1 and x2 = 4
function of x or constant, can be solved as follows cubing
Hence, Xj = 4 satisfies the original equation, but x2 = 1 does
both sides of the equation, we obtain
not satisfy the original equation.
x2 = 1 is the extraneous root. /(x) + g(x) +3 7/(x) g(x) (V7(x) + Vg(x)) = h3(x)

I Example 80. Solve the equation => /(x) + g(x) +3 7/(x) g(x) (/i(x)) =/i3(x)

37(7+3) - 7(7^2) = 7. [from Eq. (i)]


Chap 02 Theory of Equations 143

We find its roots and then substituting, then into the We get. u = 4, v = 3
original equation, we choose those which are the roots of y/2x2 + 5x - 2 = 4
the original equation. 2x2 +5x-18 = 0
Xj = 2 and x2 = - 9 / 2
I Example 82. Solve the equation
Both roots satisfies the original equation.
Vp7^i) + V(^i) = i. Hence, xt =2 and x2 = - 9 / 2 are the roots of the original
SoL We have, ^/(2x - 1) + ^(x - 1) = 1 —(i) equation.
Cubing both sides of Eq. (i), we obtain
2x - 1 + x - 1 + 3 • ^/(2x -l)(x -1) Irrational Inequations
(V(2T<) + V(x-1)) = 1 We consider, here inequations which contain the
3x - 2 + 3-^(2x2 -3x + l) (1) = 1 [from Eq. (i)] unknown under the radical sign.
3-^(2x2-3x + 1) = 3 - 3x
=> ^/(2x2 - 3x + 1) = (1 - x)
Some Standard Forms
Again cubing both sides, we obtain
to Solve Irrational Inequations
2x2 - 3x + 1 = (1 - x),3: Form 1 An inequation of the form

(2x - l)(x - 1) = (1 - x)3 2V/{*) < 2ylg(x)> nE N


=> (2x - l)(x - 1) =-(x - l)3 /W>0
is equivalent to the system
=> (x -1) {2x - 1 + (x - I)2} = 0 g(x)>f(x)
=> (x-l)(x2) = 0 and inequation of the form 2n+^Jf(x) <2n^g(x), nE N
X] = 0 and x2 = 1 is equivalent to the inequation/(x) <g(x).
X] = 0 is not satisfies the Eq. (i), then X! = 0 is an
extraneous root of the Eq. (i), thus x2 = 1 is the only root of I Example 84. Solve the inequation
the original equation.
3 7 6
Form 4 An equation of the form 5, -------- F-------
x+ 1 x+2 X-1
tfa-f(x) + y/b+f(x) = #(x).
Sol. The given inequation is equivalent to
Let u = yja-f(x),v = ^Jb + f(x) 3 +-----
7 <-----
6
----
Then, the given equation reduces to the solution of the x+1 x+2 x-1
system of algebraic equations. 4x2 - 15x - 25
=> (x + l)(x + 2)(x - 1) <
u + v = g(x)
un + v" = a + b (x + 5 / 4)(x - 5) <Q
=>
(x + l)(x + 2)(x - 1)
i I Example 83. Solve the equation
i From Wavy Curve Method :
I
l 7(2x2 + 5x-2) - 72x2 + 5x-9 =1.
■ Sol. Let u = 7(2x2 + 5x - 2)

and v = 7(2x2 + 5x - 9)
,-2)ul —,1 o(l,5)
u2 =2x2 +5x -2 k 4 /
and v2 =2x2 +5X-9 Form 2 An inequation of the form
Then, the given equation reduces to the solution of the 2!ffM <gM,n£ N.
system of algebraic equations.
u-v=1 fW>0
u2-v2=7 is equivalent to the system gW>o
(u + v)(u - v) = 7
fW<g2nM,
u+v=7 [vu-v = l]
144 Textbook of Algebra

and inequation of the form 2n+7/(x) < g(x), n G N


Exponential Equations
is equivalent to the inequation /(x) < g2r,+1(x). If we have an equation of the form a x =b(a>0), then

i Example 85. Solve the inequation ,J(x + 14) < (x + 2). (i) x G 0, if b < 0
(ii) x = logfl b, if b > 0, a * 1
Sol. We have, -J(x + 14) < (x + 2)
(iii) xG(J),ifa = l, 6*1
This inequation is equivalent to the system
(iv) x 6 R, if a = 1, b = 1 (since, lx = 1 => 1 = 1, x G R)
x + 14 > 0 x £ -14
x +2>0 x > -2 I Example 87. Solve the equation
,2
■ 14 < (.v + 2) x2 + 3x - 10 >0 7(6 - x) (3x2-7-2x+3-9 -973) = o.
x > -14 x>-14 Sol. We have,
x>—2 => x > -2 7(6 - x) (3x2~7‘2x + 3'9 - 9^3) = 0
(x + 5)(x-2)>0 x < - 5 and x > 2
This equation is defined for
On combining all three inequation of the system, we get 6 - x > 0 i.e., x < 6 ...(i)
x > 2, i.e. x G (2, co) This equation is equivalent to the collection of equations
76 - x = 0 and 3x2-72x+3-9 - 9^ = 0
Form 3 An inequation of the form
24fM>g(x),neN x,=6 and 3x2_7-2x + 3-9 = 32.5

is equivalent to the collection of two systems of then x2 - 7.2X + 3.9 =2.5


inequations x2-7.2x + 1.4 = 0
g(x)>0 . g(x)<Q
i.e. and < We find that, x2 = -1 and x3 = 7
f(x)>g2n(x) l/(x)>0
Hence, solution of the original equation are
and inequation of the form 2n +^f(x) > g(x),ne N
[which satisfies Eq. (i)J
is equivalent to the inequation /(x) >g2n+1(x). , 1
Xj = 6, x2 =

I Example 86. Solve the inequation


a/(-x2 + 4x - 3) > 6 - 2x.
Some Standard Forms to
Solve Exponential Equations
Sol. We have, yj(-x22 + 4x - 3) > 6 - 2x
Form 1 An equation in the form = 1, a > 0, a t1
This inequation is equivalent to the collection of two is equivalent to the equation /(x) =0
systems, of inequations
6 - 2x > 0 6-2x <0 B Example 88. Solve the equation 5x2+3x+2 =1.
i.e. and
-x2 + 4x - 3>(6-2x)z -x2 + 4x-3>0
Sol. This equation is equivalent to
x<3 x>3 x2 + 3x + 2 = 0
and
(x - 3) (5x - 13) < 0 (x-l)(x-3)<0 => (x + l)(x + 2) = 0
x<3 x>3 :. X] = -1, x2 = - 2 consequently, this equation has two
13 and roots X! = -1 and x2 = - 2.
— < x <3 1 < x<3
5 Form 2 An equation in the form
The second system has no solution and the first system has
fl3 A /(ax)=0
solution in the interval — < x < 3 .
k5 J is equivalent to the equation/(t) =0, where t = ax.
(13 )
Hence, x gI —, 3 I is the set of solution of the original
If tk are the roots of/(f) =0, then
inequation.
ax -tx1»,ax ~t?2»,ax =t3,...,ax = tk
Chap 02 Theory of Equations 145

I Example 89. Solve the equation 52x-24-5x -25 = 0. Let t = 2X,


Then, 30t2 - 135t + 60 = 0
Sol. Let 5X = t, then the given equation can reduce in the form
6t2 -27t + 12 = 0
t2 -24t -25 = 0
6t2 -24t -3t + 12 = 0
=> (t-25)(t + l) = 0 => t*-l,
(t-4)(6t-3) = 0
t =25,
then 5X = 25 = 52, then x = 2 Then, t1 = 4 and t2 = -
Hence, X] = 2 is only one root of the original equation. Thus, given equation is equivalent to
Form 3 An equation of the form
2X = 4 and 2X = -
aafM + p^(x) + yc/(x) =0, 2
Then, x}=2 and x2 = -1
where a, 0, y G R and a, 0, y / 0 and the bases satisfy the
Hence, roots of the original equation are Xj = 2 and
condition b2 = ac is equivalent to the equation
x2 = -1.
at2 + pt + y = 0, where t = (a / b) ^(x> Form 5 An equation of the form =c,
If roots of this equation are t, and t2, then where a,b,cE R and a, b,c satisfies the condition
(a / b)^x^ = tj and (a / b)-^(x) = t2 a2 + b2 = c, then solution of this equation is /(x) = 2 and
no other solution of this equation.
I Example 90. Solve the equation
64-9x -84-12* + 27-16* =0. 3 Example 92. Solve the equation 3x-4
X"4 + 5X-Z| = 34.
Sol. Here, 32 + 52 = 34, then given equation has a solution
Sol. Here, 9x16 = (12)2.
x-4=2.
Then, we divide its both sides by 12x and obtain x1 = 6 is a root of the original equation.
X X
3 4
=> 64-
4
I -84 + 27-
3
I =o (i) Form 6 An equation of the form {/(*)}* ^ is
X equivalent to the equation
3
Let = t, then Eq. (i) reduce in the form =101<x)log/,x’,
4
64t2 -84t + 27 =0 where f(x) >0.

3 and□ t2 = —
9 1 Example 93. Solve the equation 5X Vi
t, = — = 500.
4 16
X X 2 Sol. We have, 5X^8X-1 = 53-22
3 3 i1 f33 3
then, and —
4 4 4 4
5X -8k x ' = 53-22
X] = 1 and x2 = 2 3x-3
Hence, roots of the original equation are Xj = 1 and x2 = 2. => 5x-2 x = 53-22
Form 4 An equation in the form
a-a/(x) + P- bf^ +c = Q, 5x-3-2 1
=> (521/x)(x-3) = 1
where a, 0, c G R and a, 0, c * 0 and ab = 1 (a and b are
inverse positive numbers) is equivalent to the equation is equivalent to the equation
.(x-3)log(5-2
1Q1X- jpogp-i 1/jt ) _ j
at2 + ct + p = 0, where t = .

If roots of this equation are tj and t2, then = tj and => (x - 3) log (5-2Vx) = 0
</(x)=t2. Thus, original equation is equivalent to the collection of
equations
I Example 91. Solve the equation x-3 = 0, log(5-2,/x) = 0
1
15-2X+1+15-22-x =135. Xi =3,5-21/x = 1 => 2,1/x
1
5
Sol. This equation rewrite in the form x2 = - logs 2
Hence, roots of the original equation are Xj = 3 and
30.2X + — = 135
2X x2 = — log5 2.
146 Textbook of Algebra

Exponential Inequations where a, P, y G .R and a, P, y * 0 and the bases satisfy the


condition b2 = ac is equivalent to the inequation
When we solve exponential inequation
at2+Pt + y>0 or at2+pt + y<0,
> b (a > 0), we have
where t =(a/b)^x\
(i) XG Dj, if b <0
(ii) If b > 0, then we have /(x) > loga b, if a > 1 Form 3 An inequation of the form
and /(x) < loga b, ifO <a < 1 for a = 1, then b < 1. a</(x) + pi7(x) +y>0

or a/(x) +pf/(x) +y<0


Remark
1 < b has no solution for b < 0, a > 0, a * 1.
The inequation ar/(x) where a, P, y G R and a, P, y / 0 and ab = l(a and b are
inverse (+ve) numbers) is equivalent to the inequation
8 Example 94. Solve the inequation 3X + 2
2 1/X
at2+pt + y>0 orat2+pt + y<0
9
>3-2,x
where t=
Sol. We have, 3X + 2 > (3-2)1 /x 3X + Z
Form 4 If an inequation of the exponential form reduces
to the solution of homogeneous algebraic inequation, i.e.
aj" (x) +«,/”■'« g(x) + a2/"-2(x)g2(x) + ...

Here, base 3 > 1 +a -JWg"-'(x} + anS,'W>0,


x2 + 2x + 2
x+2>-— => >0 where a0,a1)a2, are constants (a0 ^0) and /(x)
x x and g(x) are functions of x .
(X + l)2 + 1
=> >0 => x > 0
X I Example 96. Solve the inequation
X 6 (0, oo) 22x2-10x+3 ^gx2-5x+1 > 32x2-10x + 3

Sol. The given inequation is equivalent to


Some Standard Forms to Solve g 22(x2-5x) +6-2x2-5x .3x2"5x _ 27 3,2: (x2-5x)>0

Exponential Inequations Let 2x2’5x=f(x) and 3x2"5x=g(x),


Form 1 An inequation of the form
then 8-f2(x) + 6/(x)-g(x)-27g2(x)>0
/(ax)>0 or/(ax)<0
On dividing in each by g2(x) [••' g(x)>0]
is equivalent to the system of collection
t > 0, where t = a x
Then, 8 2 + 6 [W -27 >0
sM)
f(t)>0 or /(t)<0 and let
fw_t [vf>0]
g(x)
I Example 95. Solve the inequation then 8t2 + 6t -27 >0
4X+1-16X <2logz;8. 3^
t -- (t + 9/4) >0
Sol. Let 4X = t, then given inequation reduce in the form 2/
f >3/2 and t <-9/4
4t - t2 >2- [vt>0]
2 The second inequation has no root.
=> tz-4t+3<0 => (t-l)(t-3)<0 From the first inequation, t > 3 / 2
x2-5x
1 < t <3 [vt>0] 2
> •;?<1
1 < 4X <3 3 3J 3
0 < x < log4 3 => x2 -5x<-l => x2 - 5x + l<0
x G (0, log4 3)
5 - V21 5 + 721
Form 2 An inequation of the form <x<
2 2
aa^x) +pb-^(x) +yc-^(x) >0
5-' 51 5 + 751
Hence, xg
aa^x) +p/>-f(x) +yc^(x) <0 2 ’ 2
or
Chap 02 Theory of Equations 147

Logarithmic Equations § Example 99. Solve the equation log(|Og5 x) 5 = 2.

If we have an equation of the form Sol. We have, log0og5 x)5 = 2

logo f(x) = b,(a>0),a*l Base of logarithm > 0 and 1.

is equivalent to the equation log5 * > 0 and logs* * 1

/(x) = ? (/(x)>0). => x > 1 andx *5

I Example 97. Solve the equation The original equation is equivalent to


log3(5+4log3(x-l)) = 2. log5x = 51/2 = 75
Sol. We have, log3(5 + 4 log3(x - 1)) = 2
x -5^
is equivalent to the equation (here, base # 1, > 0).
5 + 4log3(x - 1) = 32 Hence, 5^ is the only root of the original equation.

=> log3(x-l) = l => x - 1 = 3* Form 2 Equations of the form


x=4 (i) /(loga x) =0, a >0,a ^land
Hence, Xj = 4 is the solution of the original equation.
(ii) g(logx A)=0,A>0
Then, Eq. (i) is equivalent to
Some Standard Formulae to Solve f(t) =0, where t - loga x
Logarithmic Equations If *k 316 the roots of/(t) =0, then
f and g are some functions and a > 0, a 1, then, if
loga x = ti,loga x = t2,...,loga x = tk
f > 0, g > 0, we have
and Eq. (ii) is equivalent to /(y) =0, where y = logx A.
W loga(/g) = loga / + loga g
Ify1.y2.T3>---> yk are the roots of f(y) =0, then
(ii) loga(/7g) = loga /-loga g
logx A = yj,logx A = y2,-.,logx A = yfc
(iii) loga /2a = 2aloga \f\ (iv) logap f0' =
t10g“z I Example 100. Solve the equation
(v)/log^ =gloga^ (vi) a10ga/ =f l-2(logx2)2_
= 1.
log X-2 (log X) 2
H Example 98. Solve the equation
2xlog4 + 3|084 x =27. Sol. The given equation can rewrite in the form
1-2(2 log x)2 = i
Sol. The domain of the admissible values of the equation is
logx-2 (logx)2
x > 0. The given equation is equivalent to
2 3i°g4 x + 3l°g i *' = 27 [from above result (v)] 1 -8(logx)2
-1 = 0
=> 3.3log,x -27 logx -2(logx)2
3>°g4 x = 9 Let logx = t,
=> 3log4 X = 32 1 —8r2 , n 1 - 8t2 - t + 2t 2
then 2 - 0 => —=0
=> log4 x = 2 t -2t2 t -2t2
=> X] = 42 = 16 is its only root. l-t-6t2 (1 4-20(1 - 3t) _
=> = 0 => 0
(t-2t2) 01-20
Some Standard Forms to
Solve Logarithmic Equations 1
t = --
,
logx = --
1
Xj = 10-1/2
=> 2 z =>
Form 1 An equation of the form logx a = b, a > 0 has 1 , 1 x2 = 101/3
t=- logx = -
(i) Only root x = a1 ,b, if a * 1 and b = 0. 3
(ii) Any positive root different from unity, if a = 1 and b = 0. Hence, x1 = ~^= and x2 = V10 are the roots of the original
(iii) No roots, if a = 1, b 0. 10
equation.
(iv) No roots, if a * 1, b = 0.
148 Textbook of Algebra

I Example 101. Solve the equation x<1


X X

log * 10 — 6 log * 10 +11 log x 10-6 = 0. 1


I -3 I +1 = o
2 l2>
Sol. Put logx 10 = t in the given equation, we get
t3 -6t2 + lit -6 = 0 => (t-l)(t-2)(t-3) = 0, X<1
i T =>
t=1 = 3,1- + 1*0 x = (— log23)
2 <2
then t =2
Hence, Xj = - log2 3 is the root of the original equation.
t =3
It follows that Example 103. Solve the equation logr2 + x> 7 = 10g(_J_]7,
I io J (x + lj
logx 10 = 1 X= 10 x = 10
■ logx 10 = 2 => x2 = 10 => x = 710 [v x >0and* 1] Sol. The given equation is equivalent to
logx 10 = 3 x3=10 x = 7io
[v x > Oand * 1] ?7>0 x + 1 >0
/. X] = 10, x2 = 710 and x3 = 710 are the roots of the
original equation.
■ St*1 x*1
x = -6,3
2+x _ 2
Form 3 Equations of the form
10 x+1
(i) loga f(x) = loga g (x), a > 0, a * 1 is equivalent to
X! = 3 is root of the original equation.
two ways.
Form 4 Equations of the form
g(x)>0
Method I (i) logy(x) g(x) = logy(x) h(x) is equivalent to two
f(x) = g(x)
ways.
/(x)>0 g(x)>0 h(x) >0
Method II ■
/(x)=g(x) /(x)>0 /(x)>0
Method I Method II
(ii) logy(x) A = logg(x) A, A > 0 is equivalent to two /(x)^l
ways. g(x) = /i(x) g(x)=/i(x)
g(x)>0
(ii) l°gs (x) /(*) ~ l°g/i(x) /(*)is equivalent to two
Methodi- g(x)*l
ways.
/(x)=g(x)
/(x)>0
/(x)>0 g(x)>0
Methodi ■
Method II - g(x)*l
/(x)=g(x) g(x) = /i(x)
' /(x)>0
I Example 102. Solve the equation
h(x)>0

Vu.2 x
2 X
Method II
-1 -4 h(x)*l
4
,g(x) = h(x)
Sol. The given equation is equivalent to
I Example 104. Solve the equation
-l>0 |OS(X2 -II(x’ + 6) = log(x2 -1)(2*2 + 5X>'
12,
21 I -1 = 2
X

-4 Sol. This equation is equivalent to the system


(2 4 5 ,
2x2 + 5x > 0 x < — and x > 0
1 |X>1 2
x2-l>0 x < -1 and x > 1
2 ' 2 x2-1*1
2x X
x * ± 72
T 3
x3 +6 = 2x2 +5x x = -2,l,3
-2| - I -3 = 0
2 12
Hence, xt = 3 is only root of the original equation.
Chap 02 Theory of Equations 149

I Example 105. Solve the equation I Example 107. Solve the equation
lo8|x!+6| <x2-1> = I°8(2xWx2-1)- 2log2x = log (7x-2-2x2).
Sol. This equation is equivalent to Sol. This equation is equivalent to the system
x2-1>0 2x > 0
2x2 + 5x > 0 (2x)2 = 7x - 2 - 2x2
2x2 + 5x * 1 x >0
x 3 + 6 = 2x2 + 5x 6x2 -7x + 2 = 0
x < -1 and x > 1 x >0
x < —5 and. x > 0 (x - 1 / 2)(x - 2/ 3) = 0
=> 2
-5 ±733 x = l/2
X # =>
4 x = 2/3
* = -2,1,3
Hence, x, = 1 / 2 and x2 = 2 / 3 are the roots of the original
Hence, Xj = 3 is only root of the original equation. equation.
Form 5 An equation of the form
Form 7 An equation of the form
l°g/i(x) (l°g^(x) f(x)) = 0 is equivalent to the system
(2m +1) loga f (x) = loga g (x), a>0,a*l, mzN
h(x) >0
g(x)>0
h(x)*l is equivalent to the system y 2m+ 1
(x) = g(x)’
■ g(x)>0
g(x)*l I Example 108. Solve the equation
f(x)=g(x) log (3x2 + x - 2) = 3log (3x - 2).

I Example 106. Solve the equation Sol. This equation is equivalent to the system
lo8?-6x+8Oo82x’-2x+s(x2 + 5x» = 0- 3x2 + x-2>0
3x 2 + x - 2 = (3x - 2)3
Sol. This equation is equivalent to the system
(x — 2 / 3) (x — 2) > 0
x2 - 6x + 8 > 0
(x -2/3) (9x2 -13x + 3) = 0
x 2 - 6x + 8 * 1
2x2 - 2x - 8 > 0 x < 2 / 3 and x > 2
2x2 - 2x - 8 * 1 2 13 ±761
x = -, x =----------
2
x2 + 5x = 2x2 - 2x - 8 3 18
Solve the equations of this system 13-V61
Original equation has the only root X] =
18
x < 2 and x > 4
x*3± 72 Form 8 An equation of the form

1- 717 , 1 + 717 loga f (x) + loga g(x) = loga m(x), a > 0, a * 1


and xx>
x < — ------and >
2------------------ 2 is equivalent to the system
x* f(x)>0
2
x = -1,8 g(x)>0
x = -1, does not satisfy the third relation of this system. f(x) g(x)=m(x)
Hence, Xj = 8 is only root of the original equation.
Form 6 An equation of the form I Example 109. Solve the equation
2m l°ga f(x) = loga g(x), a > 0, a * 1, m e N is 2log3 x + log3(x2 - 3) = log3 0.5+ 5l08s(l°83 8)
equivalent to the system Sol. This equation can be written as
/(x)>0 log3 x2 + log3(x2 - 3) = log3 0.5 + log38
\f2mM=g(x)
log3 x2 + log3(x2 - 3) = log3(4)
150 Textbook of Algebra

This is equivalent to the system


x2 > 0
Logarithmic Inequations
x < 0 and x > 0
When we solve logarithmic inequations
’ • x2-3>0 x < - -73 and x> y/3
loga /(x)>loga g(x)
x2(x2 -3)= 4 (x2 -4)(x2+ l) = 0
(0 a>1
x2 - 4 = 0 x = ± 2, but x > 0
g(x)>0
Consequently, X] = 2 is only root of the original equation. > • a>1
Form 9 An equation of the form f(x)>g(x)
log a/(x) -loga g(x) = loga h(x) - loga t(x), a > 0, a * 1
log0 fU)>loga g(x)
is equivalent to the equation (ii)
0 <a <1
logfl f(x) + loga t(x) = loga g(x) + loga /i(x),
which is equivalent to the system /(x)>0

/(x)>0 0 <a <1

t(x) > 0 /(x)<g(x)

g(x)>0
I Example 111. Solve the inequation
/i(x)>0 log2K+jX2 <log2x+3(2x+ 3).
/(x)-t(x)=g(x)-h(x)
Sol. This inequation is equivalent to the collection of the
systems
I Example 110. Solve the equation
2x + 3> 1 x> -1
sin — x2 <2x + 3 (x-3)(x + l)<0
log2(3 - x) - log2 __ 4_ = l + log2(x + 7). => 3
5-x 0<2x + 3<l — < x < -1
2
\ / x2 >2x +3
(x -3)(x + l)>0
Sol. This equation is equivalent to x > -1
( . 3tt A => — 1 < x < 3
sin — -l<x<3
log2(3 - x) = log2 4 + ^log22 + log2(x + 7) 3
5-x — < x < -1 3
2 => -- < x < -1
\ > x < -1 and x > 3 2
/
=> log2(3 - x) = log2
1 ] + log2 y[2 + log2(x + 7) Hence, the solution of the original inequation is
vV2(5-x)J c 3 'i
xG —,-1 U(-1,3).
which is equivalent to the system I 2 J

3-x>0 Canonical Logarithmic Inequalities


1 1 flogax>0 x >1
>0
• V2(5-x)
a>1 a>1
x +7 >0
V2(x + 7) loga x >0 0<x<l
(3 - x) = 2. <
V2(5-x) 0 <a < 1 0 <a <1

loga x <0 0 <x <1


3.
x<5
a>1 a>1
x>-7
(x-l)(x-8) = 0 logfl x <0 x >1
4J
0<a <1 0<a<l
Hence, x} = 1 is only root of the original equation.
Chap 02 Theory of Equations 151

Some Standard Forms to Solve <=>


x < 6 - V6 and x > 6 + V6 and 2 < x < 10
0< x <5
Logarithmic Inequations
Therefore, the system has solution 2 < x < 6 - ^6
Form 1 Inequations of the form
combining both systems, then solution of the original
Forms Collection of systems inequations is
(a) logg(x)> 0 [/(*)> 1,|0 < /(x)< 1 xe(2,6- V6)u(10, °o).
[g(x)> l,[0<g(x)<l
Form 2 Inequations of the form
(b) 1Qg«(x)/W^o f/(x)>l,|0</(x)<l
|g(x)> 1,10<g(x) < 1 Forms Collection of systems
(a) logo(x) /(x) > log9(x) g(x) /(x)>g(x),
(c) logg(x)/(x)<0 /(x)>l, |o</(x)<l
- g(x)>0,
0<g(x)<l,[ g(x)>l

(d) logg(x) f(x)<Q ' /(x)>l, [0</(x)<l f(x)<g(x)


0<g(x)<l,[ g(x)>l
■ jW>0
1 Example 112. Solve the inequation 0 < 0(x) <1
2x^
Jlog2— >0. (b) log^/fxj^log^gfx) '/(x)>g(x),
x2-12x4-30
10 ■ g(x)>0,

So/. This inequation is equivalent to the collection of two <Xx)>l,


systems /(x)<g(x)
x2 - 12x 4-30 ■ Z(x)>0
>1,
10 0<<f)(x)<l
logal y I > 1.
(c) log(>(x)/(x)<log(;,(x)g(x) f(x)<g(x),
x2 - 12x 4-30 < • /’(x)>0,
0<
10 <Xx)>l,
n 1 (2x^1 ,
0 < log2l — J < 1 /(x)>g(x)
• g(x)>0
On solving the first system, we have
0 < (Xx) < 1
x2 —12x + 20 >0
=> 2x (d) log«(x)/(*)^log«(x)$(*) <=> f(x)<g(x),
— >2
5 - f(x)>0,
(x - 10)(x - 2) > 0 <X*)>1.
<=>
x>5
f(x)>g(x)
x < 2 and x > 10 ’ g(x)>0
<=>
x>5 0 < <p(x) <1
Therefore, the system has solution x > 10.
On solving the second system, we have I Example 113. Solve the inequation
0 < x2 - 12x 4-30 < 10 log(X-3)(2 (x2 - lOx + 24)) > log(x_3)(x2 - 9).
=>
1 < — <2 Sol. This inequation is equivalent to the collection of systems
5
x2 - 12x 4- 30 > 0 and x2 - 12x 4- 20 < 0 2(x2 - lOx 4-24) > x2 - 9,
<=>
5/2<x<5 x2 - 9 > 0,
x - 3 > 1,
152 Textbook of Algebra

2(x2 - lOx + 24) < x2 -9 On solving the second system, we have


o o
2(x2 - lOx+ 24)>0
-o
0< x-3<1
On solving the first system, we have
x2 - 20x + 57 > 0, 3 10-743 4 6 10 +fi3
• (x + 3)(x - 3) > 0, x2 -20x + 57 <0,
(x - 6)(x - 4) > 0,
x e (-«>, 10 - V43] u [10 + V43,00) 3 < x < 4,
<=> xe (-00,-3) 0(3, 00) x G [10 - V43,10 + V43]
XG(4,oo) <=> x G (-00, 4) U (6, 00)

Therefore, the system has solution X G (3, 4)


o--------------
Therefore, the system has solution
■o o
10 - V43 < x < 4,
-3 3! 10-^43 4 10 + 743 i.e., x G [10 - V43,4)

x >10 + 5/43 On combining the both systems, the solution of the original
inequation is
i.e. X G [10 + V43, «) x G [10 - V43, 4) u [10 + V43, «>).

g Exercise for Session 5


1. The equation ^(x + 1) ~ ~ 1) = V<4x "1) has
(a) no solution (b) one solution (c) two solutions (d) more than two solutions
2. The number of real solutions of 7(x 2-4x + 3) + 7(x 2 - 9) = ^(4x2 - 14x + 6) is
(a) one (b) two (c) three (d) None of these
3. The number of real solutions of -J(3x2 -7x -30) - 7[2x7z - 7x - 5) = x - 5 is

(a) one (b) two (c) three (d) None of these


4. The number of integral values of x satisfying ^(-x2 + 10x -16) < x -2 is
(a)0 (b)1 (c)2 (d)3

5. The number of real solutions of the equation


O’
(a) 2 (b) 1 (c)0 (d) None of these
6. The set of all x satisfying 32x - 3X - 6 > 0 is given by
(a) 0 < x < 1 (b) x > 1 (c) x > 3"2 (d) None of these

7. The number of real solutions of the equation 2!x/2 + (>/2 + 1)x =(3 + 2V2)x/2 is
(a) one (b) two (c) four (d) infinite
8. The sum of the values of x satisfying the equation (31 + 8Vl5)xJ 2-3 + 1 = (32 + 8Vi5)x2'3 is
(a) 3 (b) 0 (c) 2 (d) None of these
9. The number of real solutions of the equation log0 5 x = | x | is
(a)0 (b)1 (c)2 (d) None of these
10. The inequality (x -1)In(2 - x) <0 holds, if x satisfies
(a)1<x<2 (b)x>0 (c)0<x<1 (d) None of these

You might also like